PeriOp Flashcards

1
Q

2hyp2.1A high mixed venous oxygen saturation (SvO2) is most likely to be associated with

a. COPD
b. PE / Tamponade
c. Acute MI
d. Severe liver failure
e. Sepsis

A

d. Severe liver failure
but could also be
e. Sepsis

LIFTL:

INTERPRETATION

High SvO2
- increased O2 delivery (increased FiO2, hyperoxia, hyperbaric oxygen)
- decreased O2 demand (hypothermia, anaesthesia, neuromuscular blockade)
- high flow states: sepsis, hyperthyroidism, severe liver disease

Low SvO2
- decreased O2 delivery:
1. decreased Hb (anaemia, haemorrhage, dilution)
2. decreased SaO2 (hypoxaemia)
3. decreased Q (any form of shock, arrhythmia)

  • increased O2 demand (hyperthermia, shivering, pain, seizures)
  • Causes of High SvO2 despite evidence of End-organ Hypoxia:
    1. microvascular shunting (e.g. sepsis)
    2. histotoxic hypoxia (e.g. cyanide poisoning)
    3. abnormalities in distribution of blood flow

Anesthesia Monitoring Of Mixed Venous Saturation:
https://www.ncbi.nlm.nih.gov/books/NBK539835/

In sepsis, ScvO2 less than 70% or SvO2 lower than 65% correlate with poor prognosis.[2] In application, certain studies have shown that maintaining a goal ScvO2 greater than 70% leads to reduced mortality.[11] Therefore, ScvO2 is used to guide treatment algorithms in the Surviving Sepsis Campaign (SSC).

Studies have shown that normal to higher levels of mixed venous oxygen saturation in patients with clinically worsening sepsis do not rule out tissue hypoxia due to the inability to utilize O2.[11][7] Therefore, several studies support the conclusion that abnormally low or high ScvO2 correlates with higher mortality in patients with septic shock.

How well did you know this?
1
Not at all
2
3
4
5
Perfectly
2
Q

A medication that would be acceptable to a patient who refuses all products derived
from human plasma is:

a) Prothrombinex
b) Activated factor 7
c) Fibrinogen concentrate
d) Albumin
e) anti-d

A

Factor 7 - Recombinant, made from baby hamster kidney cells

Albumin - Alburex® 5 AU (Human Albumin 50 g/L) is an Australian manufactured albumin product

Fib con - Lyophilised precipitate. manufactired from cryoprecipitate.

PCC - Prothrombinex-VF® is a lyophilised concentrate of human coagulation factors containing factors II, IX and X and a small amount of factor VII.

Red cross lifeblood.

Correct answer is rVIIa

How well did you know this?
1
Not at all
2
3
4
5
Perfectly
3
Q

Phaeochromocytoma commonly presents with all of the following EXCEPT:

a) RV Hypertrophy
b) Pulmonary HTN
c) long QT
d) ST changes
e) Cardiomyopathy

A

b) Pulmonary HTN

Long QT + ST changes common
Cardiomyopathy less common but well documented
RVH possible, although more commonly LVH

How well did you know this?
1
Not at all
2
3
4
5
Perfectly
4
Q

Organ procurement after circulatory death is generally stood down if the time from
cessation of cardiorespiratory support to circulatory death extends beyond:

a) 60min
b) 90min
c) 120min

A

90 mins

30mins
Liver
Pancreas
Heart

60mins
Kidneys

90mins
Lungs

Page 35 ANZICS statement 2.4.3 Warm ischemia time

Donate life

How well did you know this?
1
Not at all
2
3
4
5
Perfectly
5
Q

The bipolar leads of a 12-lead electrocardiogram are:

a) All
b) V1-V6
c) aVL, aVR, aVF
d) I, II, III
e) None

A

D) I, II, III

3-electrode system
- Uses 3 electrodes (RA, LA and LL)
- Monitor displays the bipolar leads (I, II and III)
Life in the Fast Lane

How well did you know this?
1
Not at all
2
3
4
5
Perfectly
6
Q

The Glasgow Blatchford score is used to risk stratify:

a) Pulmonary haemorrhage
b) Traumatic intraperitoneal haemorrhage
c) PPH
d) SAH
e) UGI bleed

A

e) UGI bleed

Stratifies upper GI bleeding patients who are “low-risk” and candidates for outpatient management. Use for adult patients being considered for hospital admission due to upper GI bleeding.

Components: haemoglobin, BUN, initial systolic BP, heart rate > 100, melena present, recent syncope, hepatic disease history, cardiac failure present.

Med-Calc

How well did you know this?
1
Not at all
2
3
4
5
Perfectly
7
Q

A single intraoperative dose of 8 mg dexamethasone compared to 4 mg results in:

a) No difference in analgesia
b) No difference in PONV
c) No difference in BSL
d) Increased surgical site infection

A

B is the answer

Check 4th consensus guidelines

Does show better analgesia

PADDI Trial (Monash and ANZCA) 2021
No difference in infection with dex 8 vs placebo

Anaesthesiology Nov 2021, Vol 135, issue 5 - article by Aus anaesthesiologists

A higher dose
- Will cause more hyperglycaemia in DM patients but not clinically/statiscally significant
- Will improve PONV for 72 hours = possibly
- Some studies show this can improve analgesia - ortho, ent cases particularly
8mg dose recommended
Was the question related to addition of dex in block - Korean study compared 4vs8 in 2018

How well did you know this?
1
Not at all
2
3
4
5
Perfectly
8
Q

The minimum age in years for in vitro contracture testing for suspected malignant
hyperthermia is

a) 6
b) 8
c) 10
d) 12

A

10

All current Australian and New Zealand laboratories follow the guidelines of the European Malignant Hyperthermia Group for In Vitro Contracture Testing.

The EMHG guidelines are summarised as follows:

Age and Weight

The minimum weight limit for Australian and New Zealand laboratories is 30 kg and the minimum age for IVCT is 10 years.
(Emhg actually says min age for muscle biopsy is 4 yrs but lab’s should not test children under 10 yrs without relevant control data)

IVCT details

The biopsy should be performed on the quadriceps muscle (eithervastus medialisorvastuslateralis), using local (avoiding local anaesthetic infiltration of muscle tissue), regional, or trigger-free general anaesthetic techniques.

The muscle samples can be dissected in vivo or removed as a block for dissection in the laboratory within 15 minutes.

The time from biopsy to completion of the tests should not exceed 5 hours.

Muscle specimens should measure 20-25 mm in length and at least four tests should be performed each one using a fresh specimen.

The tests should include a static cumulative caffeine test and a dynamic or static halothane test.

The results should be reported as the threshold concentration, which is the lowest concentration of caffeine or halothane that produces a sustained increase of at least 2 mN (0.2 grams) in baseline force from the lowest force reached.

How well did you know this?
1
Not at all
2
3
4
5
Perfectly
9
Q

Regarding sex differences in the incidence of connected consciousness (ability to
respond to command during general anaesthesia) in adults after tracheal intubation
as measured by the isolated forearm technique:

a) Higher in females due to lower propofol ml/kg dose
b) Higher in females despite same dose propofol
c) Higher in males due to lower propofol ml/kg dose
d) Higher in males despite same propofol dose
e) No sex difference

A

B) higher in females despite same dose propofol

BJA Feb 2023

https://www.bjanaesthesia.org/article/S0007-0912(22)00192-1/fulltext

Females (13%, 31/232) responded more often than males (6%, 6/106). In logistic regression, the risk of responsiveness was increased with female sex (odds ratio [ORadjusted]=2.7; 95% confidence interval [CI], 1.1–7.6; P=0.022) and was decreased with continuous anaesthesia before laryngoscopy

*supplementary table shows dosing between female and male responders vs non responders and dosing is the same

How well did you know this?
1
Not at all
2
3
4
5
Perfectly
10
Q

A drug which is unlikely to interfere with skin testing is oral:

a) Diphenhydramine
b) Amitriptyline
c) Prednisolone
d) Risperidone
e) Ranitidine

A

MAYANK Risperidone

Avoid antihistamines and steroids
TCAs known to interfere

Mayo clinic website

See allergy.org.au - risp mentioned in appendix b as a med that may need held

How well did you know this?
1
Not at all
2
3
4
5
Perfectly
11
Q

A healthy woman with an uncomplicated pregnancy has an American Society of Anesthesiologists (ASA) Physical Status classification of:

a) 1
b) 2
c) 3

A

2

How well did you know this?
1
Not at all
2
3
4
5
Perfectly
12
Q

Steph Oral naltrexone should be ceased preoperatively for:

a) 24 hours
b) 48 hours
c) 72 hours
d) 96 hours

A

NAOMI 72 hours
ANZCA Blue Book 2023

Oral naltrexone should be stopped at least 24 hours and ideally 72 hours prior to elective surgery.
And there is a lack of instruction re Contrave- so best to stop 72 hours prior.
And limited evidence re low dose naltrexone for chronic pain - so for consistency blue book says 72 hours.

Caution increased opioid sensitivity in patients using perioperative naltrexone.

How well did you know this?
1
Not at all
2
3
4
5
Perfectly
13
Q

Steph In a patient presenting with an Addisonian crisis, the electrolyte disturbances MOST LIKELY to be seen are:

a) Low BSL, hyperkalaemia, hyponatraemia
b) High BSL, hyperkalaemia, hyponatraemia
c) Hypocalcaemia, hyperkalaemia, hyponatraemia
d) Hypercalcaemia, hyperkalaemia, hyponatraemia

A

a) Low BSL, hyperkaelamia, hypernatraemia

Adrenal crisis is a medical emergency and should be considered in any patient presenting with one or more of the following symptoms:
* altered consciousness
* circulatory collapse
* hypoglycaemia
* hyponatraemia
* hyperkalaemia
* seizures
* history of steroid use/withdrawal
* any clinical features of Addison disease

Adrenal crisis may be precipitated by stress, sepsis, dehydration or trauma; clinical features may be modified accordingly. In patients with known adrenal insufficiency, nonadherence with therapy, inappropriate cortisol dose reduction or lack of stress related cortisol dose adjustment can cause adrenal crisis.

Aus Family Physician - RACGP

Re chat below - incorrect recall, have updated
A

Why A? All three should be seen - glucocorticoid deficiency causes low Na and glucose while simultaneous mineralocorticoid deficiency low K.

Crisis typically presents with hypotension abdo pain, nausea, vomiting and confusion. No one electrolyte/lab value can tie all those together.

How well did you know this?
1
Not at all
2
3
4
5
Perfectly
14
Q

A 50-year-old has had a headache for the last month which is relieved by lying flat. They have had no medical procedure to their spine such as epidural, spinal or lumbar puncture. Their brain magnetic resonance (MR) imaging scan shows diffuse
meningeal enhancement and brain sagging. The neurologist suspects spontaneous intracranial hypotension and asks you to do an epidural blood patch. No spinal imaging has been performed to confirm a cerebrospinal fluid (CSF) leak. You should

A do LP to measure pressure if low do lumbar patch
B do blood patch at lumbar level with no further investigation
C do spine imaging if CSF leak present do blood patch at level
D do spine imaging if CSF leak present do lumbar blood patch
E refuse to do blood patch

A

REPEAT

B do blood patch at lumbar level with no further investigation

How well did you know this?
1
Not at all
2
3
4
5
Perfectly
15
Q

A patient presents for a trans-urethral resection of the prostate (TURP). He had a single drug-eluting coronary stent for angina pectoris inserted six months ago and is taking clopidogrel and aspirin. The most appropriate preoperative management of his medications is to:

a) Cease aspirin, continue clopidogrel
b) Cease aspirin for 10 days, cease clopidogrel for 5 days
c) Cease clopidogrel for 5 days, continue aspirin
d) Cease clopidogrel for 10 days, continue aspirin
e) Continue both aspirin and clopidogrel

A

REPEAT

c) Cease clopidogrel for 5 days, continue aspirin
- prostatic surgery, the risk of major bleeding may be greater than the risk of stent thrombosis
- For clopidogrel, we stop five days before surgery
- Clopidogrel, if stopped, should be restarted with a loading dose of 300 mg as soon as possible after surgery, perhaps later in the day if postoperative bleeding has stopped. Some experts recommend a higher loading dose of 600 mg to decrease time to effectiveness in the higher-risk postoperative setting
- suggest that surgery be performed in centers with 24-hour interventional cardiology coverage

UP TO DATE: Noncardiac surgery after PCI

Nonemergency noncardiac surgery — For patients who have undergone previous stenting with either BMS or DES and who will need cessation of one or both antiplatelet agents, we prefer to defer planned nonemergency, nonurgent noncardiac surgery until at least six months after stent implantation. The risks of noncardiac surgery before six months are increased after both BMS and DES.
For patients whose surgery requires cessation of one or both antiplatelet agents and cannot wait six months, and where the risks of delaying surgery outweigh the benefits, our recommended minimal duration of DAPT is four to six weeks, depending on the urgency of surgery and risk of thrombotic complication. This is based in part on evidence suggesting that the increased risk of MI and cardiac death is highest within the first month after stent placement and no clear difference in risk between BMS and DES. Although we prefer to wait at least six weeks when possible, in patients for whom earlier surgery is in their best interest after weighing risks and benefits, we sometimes refer patients as early as four weeks after stent placement.

The proinflammatory and prothrombotic risks of surgery may increase the baseline risk of stent thrombosis even in the presence of DAPT and regardless of stent type during this early period after stenting. We believe this risk to be higher prior to the minimum duration of DAPT recommended above, but the final decision to continue or discontinue antiplatelet therapy in the perioperative period should be made only after an informed discussion among the surgeon, managing cardiologist (and other health care providers), and patient has taken place. In many cases, DAPT can be continued in the perioperative period, although for some surgeries, such as neurosurgery, posterior eye surgery, or prostatic surgery, the risk of major bleeding may be greater than the risk of stent thrombosis.

In these patients who undergo noncardiac surgery before the recommended minimum duration of DAPT, a platelet P2Y12 receptor blocker should be discontinued for as brief a period as possible. Aspirin should be continued through the perioperative period, since the risk of stent thrombosis is further increased with the cessation of both aspirin and clopidogrel and surgery can usually be safely performed on aspirin. The rationale to continue aspirin comes in part from the POISE-2 trial (PCI subgroup analysis), which is discussed separately. However, as many neurosurgical patients, for whom bleeding might be life threatening or lead to severe adverse outcomes, were not enrolled in POISE-2, the optimal strategy is not known.

●Minor surgical and dental procedures usually do not require cessation of antiplatelet therapy.
●With regard to stopping P2Y12 inhibitor prior to noncardiac surgery, we generally follow recommendations found in the manufacturer’s package insert for each drug.
- For clopidogrel, we stop five days before surgery; that is, the last dose is taken on the sixth day before surgery.
- For prasugrel, we stop seven days before surgery.
- For ticagrelor, we stop three to five days before surgery.
- Some experts are willing to recommend shorter discontinuation periods for procedures less likely to be associated with major bleeding.
●Clopidogrel, if stopped, should be restarted with a loading dose of 300 mg as soon as possible after surgery, perhaps later in the day if postoperative bleeding has stopped. Some experts recommend a higher loading dose of 600 mg to decrease time to effectiveness in the higher-risk postoperative setting.
●We suggest that surgery be performed in centers with 24-hour interventional cardiology coverage

How well did you know this?
1
Not at all
2
3
4
5
Perfectly
16
Q

According to the 5th National Audit Project (NAP5), the incidence of awareness during general anaesthesia using a non-relaxant technique with a volatile agent is approximately:

a. 1:700
b. 1:8000
c. 1:10000
d. 1:19000
e. 1:136,000

A

REPEAT
e. 1:136,000

https://www.bjanaesthesia.org/article/S0007-0912%2817%2930746-8/fulltext

1/670 E-LSCS
1/8000 with muscle relaxation
1/8600 CTS
1/8200 Volatile + neuromuscular blocking
Overall 1:19000

How well did you know this?
1
Not at all
2
3
4
5
Perfectly
17
Q

A patient for elective general anaesthesia has been noted to be chewing gum in the pre-operative area. The most appropriate course of action is to:

a) Delay 1 hour
b) Delay 2 hours
c) Delay 6 hours
d) Proceed
e) Cancel

A

d) Proceed

ANZCA PG07 appendix 1 - Chewing gum and boiled sweets should be discarded prior to inducing anaesthesia to avoid them being Inhaled as a foreign body but do not constitute an indication for delaying any procedure unless they have been ingested.

Therefore D

How well did you know this?
1
Not at all
2
3
4
5
Perfectly
18
Q

NP A 74-year-old presents for a femoral popliteal artery bypass procedure for peripheral limb ischaemia. Regarding its role in modifying their perioperative cardiovascular risk, clonidine:

a. Increased stroke
b. No change in complications
c. Increased death
d. Increased non fatal MI
e. Increased risk of non fatal cardiac arrest

A

REPEAT

e. Increased risk of non fatal cardiac arrest

POISE II
* clonidine 200mcg per day - did not reduce the rate of composite outcome of death or nonfatal MI - but it increased the risk of clinically important hypotension and nonfatal cardiac arrest
* aspirin initiation or continuation – no significant effect on rate of composite of death or non fatal MI but increased risk of major bleeding

Clonidine, as compared with placebo, was associated with an increased rate of nonfatal cardiac arrest

POISE 2 TRIAL

How well did you know this?
1
Not at all
2
3
4
5
Perfectly
19
Q

NP A drug that is contraindicated for a patient with a history of heparin induced thrombocytopaenia is:

a) Bivalirudin
b) Danaparoid
c) Prothrombinex
d) Fib conc

A

B) prothrombinex.

Prothrombinex product information states don’t give if hx of HITS

PROTHROMBINEX CONTAINS HEPARIN

How well did you know this?
1
Not at all
2
3
4
5
Perfectly
20
Q

A patient with known suxamethonium allergy is most likely to demonstrate cross reactivity with:

a. Mivacurium
b. Cisatracurium
c. Atracurium
d. Rocuronium
e. Cephazolin

A

REPEAT

AT - Rocuronium

Anaphylaxis to neuromuscular blocking drugs: incidence and cross-reactivity in Western Australia from 2002 to 2011 by Sadleir et al

(This paper was referenced in NAP 6 “Cross-sensitivity, based on skin testing and specific IgE, is common, with suxamethonium being the most commonly crossreacting drug (Sadleir 2013).”)

Fig 4 shows Rates of cross-reactivity for patients diagnosed with anaphylaxis according to the triggering NMBD.
- for sux anaphylaxis: highest cross-reactivity was roc (24%), then interestingly vec and cis were both tied at 12%, as were panc and atrac at 6%

PREVIOUS NOTES:

BJA Anaphylaxis to neuromuscular blocking drugs: incidence and cross-reactivity in Western Australia from 2002 to 2011
https://academic.oup.com/bja/article/110/6/981/245571

Rocuronium has a higher rate of IgE-mediated anaphylaxis compared with vecuronium, a result that is statistically significant and clinically important.

Cisatracurium had the lowest rate of cross-reactivity in patients who had previously suffered anaphylaxis to rocuronium or vecuronium.

Anaphylaxis rates (highest to lowest)
Primary anaphylaxis: rocuronium > atracurium > vecuronium > pancuronium = cisatracurium
Cross-reactivity: suxamethonium > rocuronium > vecuronium > pancuronium > atracurium > cisatracurium

How well did you know this?
1
Not at all
2
3
4
5
Perfectly
21
Q

The correct blood collection tube for a mast cell tryptase test is a:

a. Potassium EDTA
b. serum separating tube
c. sodium citrate
d. sodium oxalate something

A

REPEAT

b. serum separating tube (gold top tube or red)

Potassium EDTA (purple)
-> FBC

sodium citrate (blue)
-> clotting screen/Rotem

sodium oxalate (green)
-> heavy metals (lead copper zinc)

How well did you know this?
1
Not at all
2
3
4
5
Perfectly
22
Q

A new antiemetic reduces the risk of post-operative vomiting by 20%. In a population with a baseline risk of post-operative vomiting of 10%, the number needed to treat is:

a. 2
b. 5
c. 10
d. 20
e. 50

A

AT
REPEAT

(base rate is 10%, experimental group is 8% (20% below 10%) therefore 100/ 2 = 50
or 1 divided by risk reduction

population risk = 10/100 patients get PONV
population risk + new antiemetic = 8/100 patients get PONV (8/100 as reduction by 20% with new drug)

RR= 0.10-0.08=0.02
NNT= 1/RR
=1/0.02
=50

How well did you know this?
1
Not at all
2
3
4
5
Perfectly
23
Q

A risk factor which increases the likelihood of developing local anaesthetic systemic toxicity is:

a) Hypoxia
b) Alkalaemia
c) High alpha1-acid glycoprotein
d) Hypocarbia
e) Increased carnitine levels

A

AT
REPEAT

b) Hypoxia

Hypoxia

Local anaesthetics are bases with pKa above physiological pH. The more alkalaemic the environment the more unionionised (B) form there is – which will speed diffusion across plasma membrane = can exert Na+ channel blockade.

https://www.bjanaesthesia.org/article/S0007-0912(17)38238-7/pdf
https://academic.oup.com/bjaed/article/15/3/136/279390
https://www.ncbi.nlm.nih.gov/pmc/articles/PMC6087022/

Hypoxia – metabolic acidosis = ion trapping = increased toxicity
Alkalaemia = prevents ion trapping in tissues (intralipid can work) = reduced toxicity
High a1GP = reduced free fraction (a1gp high affinity, low capacity) = reduced toxicity
Low CO2 = alkalosis = prevents ion trapping in tissues (intralipid can work) = reduced toxicity
Carnitine deficiency = increased toxicity, therefore increased carnitine will reduce toxicity https://pubmed.ncbi.nlm.nih.gov/19849674/

a. Hypoxia - Yes
b. Alkalemia - No - acidosis causes increased ionised fraction due to its weak base properties
c. High α1-acid glycoprotein - No, normally bound to alpha-1 acid glycoprotein
d. Hypocarbia < (decreased seizure threshold) - No - hypercarbia increases CNS blood flow and increases risk of seizures due to more LA delivered to CNS
e. Increased carnitine levels -s - Never heard of it

How well did you know this?
1
Not at all
2
3
4
5
Perfectly
24
Q

The image below shows results from non-inferiority trials. The trial labelled ‘M’ is best described as:

a) Non inferiority graph (line crossed the 0 line but not non-inferior dotted line)

A

NIKKI

How well did you know this?
1
Not at all
2
3
4
5
Perfectly
25
Q

Analysis of variance (ANOVA) is a statistical test to determine:

a) The validity between an expected and observed outcome in a population
b) The difference between the means of more than two populations
c) The difference between two populations with non-parametric data
d) The degree of similarity of the median between two or more populations
e) If the variance within a population is likely to be abnormally or normally distributed

A

REPEAT

B) analyse the difference between the means of more than two groups

How well did you know this?
1
Not at all
2
3
4
5
Perfectly
26
Q

A third heart sound at the apex may be heard in:

a) Healthy people aged less than 40
b) Mitral prolapse
c) HTN

23.1 OPTIONS:

a) pulmonary stenosis
b) pulmonary hypertension
c) pericarditis
d) pregnancy

A

AT

Repeat

Can occur in healthy young people

The third heart sound is mainly created by the early-diastolic rapid distension of the left ventricle that accompanies rapid ventricular filling and abrupt deceleration of the atrioventricular blood flow

S3 may be normal in people under 40 years of age and some trained athletes but should disappear before middle age. Re-emergence of this sound late in life is abnormal[5] and may indicate serious problems such as heart failure.

‘Sounds like Ken-tu-cky’

How well did you know this?
1
Not at all
2
3
4
5
Perfectly
27
Q

In pulmonary function testing the presence of airflow limitation is defined by a post- bronchodilator FEV1/FVC ratio less than:

a) 0.5
b) 0.6
c) 0.7
d) 0.8

A

c) 0.7

How well did you know this?
1
Not at all
2
3
4
5
Perfectly
28
Q

Regarding healthcare research, the PICO framework describes:

a) Critical appraisal
b) Meta-analysis
c) Observational study
d) Systematic review

A

REPEAT

a) Critical appraisal

PICO is a mnemonic used to describe the four elements of a good clinical foreground question:

P = Population/Patient/Problem - How would I describe the problem or a group of patients similar to mine?

I = Intervention - What main intervention, prognostic factor or exposure am I considering?

C = Comparison - Is there an alternative to compare with the intervention?

O = Outcome - What do I hope to accomplish, measure, improve or affect?

How well did you know this?
1
Not at all
2
3
4
5
Perfectly
29
Q

The 12-lead electrocardiogram shown is most consistent with acute total occlusion
of the:

a) LAD
b) PDA
c) OM
d) RCA

A

VICTORIA

Am I missing something? I can’t see total occlusion of anything here.

Wellens syndrome- Lad

How well did you know this?
1
Not at all
2
3
4
5
Perfectly
30
Q

A 70-year-old patient booked for a revision total hip replacement is reviewed in preadmission clinic ten days before surgery. The following blood test results are

noted:
haemoglobin 110 g/L
ferritin 51 mcg/L
CRP (c-reactive protein) 10 mg/L

The most appropriate management for this patient should be to:

a) Proceed
b) Give PO iron and delay 6 weeks
c) Give IV iron
d) Blood transfusion pre-op

A

Victoria

Screenshot sent to JJ

B

How well did you know this?
1
Not at all
2
3
4
5
Perfectly
31
Q

A 55-year-old with no past history of ischaemic heart disease is three days post-total hip replacement surgery. They have an episode of chest pain at rest with features typical of angina that lasts 30 minutes before fully resolving. There are no electrocardiogram changes and no troponin rise. The diagnosis is

a. No diagnosis made
b. Unstable angina
c. STEMI
d. NSTEMI
e. MINS

A

REPEAT
b. Unstable angina

UTD:

Unstable angina (UA) and acute non-ST elevation myocardial infarction (NSTEMI) differ primarily in whether the ischemia is severe enough to cause sufficient myocardial damage to release detectable quantities of a marker of myocardial injury (troponins):

●UA is considered to be present in patients with ischemic symptoms suggestive of an ACS and no elevation in troponins, with or without electrocardiogram changes indicative of ischemia (eg, ST segment depression or transient elevation or new T wave inversion).

●NSTEMI is considered to be present in patients having the same manifestations as those in UA, but in whom an elevation in troponins is present.

MINS: Myocardial injury after non-cardiac surgery (up to 30 days post-op):
1. Elevated postop troponin
2. Resulting from myocardial ischaemia (i.e. no evidence of a non-ischaemic aetiology), not requiring an ischaemic feature (i.e. no chest pain, no ECG change)

VISION studies (Vascular Events in Noncardiac Surgery Patients Cohort Evaluation) demonstrated that severity of MINS strongly associated with 30-day mortality after NCS.

hs-cTnT
<20ng/L ~ 0.5% 30 day mortality
20-64ng/L ~3% 30 day mortality
65-999 ng/L ~9% 30 day mortality
>1000ng/L ~30% 30 day mortality

Whilst VISION trial identified MINS in at risk patients, the question now becomes what interventions are available to prevent this complication?

How well did you know this?
1
Not at all
2
3
4
5
Perfectly
32
Q

Kate
One metabolic equivalent (1MET) is defined as the:

a) O2 consumption during walking 4km/h
b) O2 consumption at rest
c) Energy consumption while walking at 4km/h
d) Energy consumption during rest

A

REPEAT

B) 02 consumptiom at rest

One metabolic equivalent (MET) is the amount of oxygen consumed while sitting at rest, equal to 3.5 mL O2 per kg body weight × min

How well did you know this?
1
Not at all
2
3
4
5
Perfectly
33
Q

Kate A bleeding patient has ROTEM results including: [table attached]. The most
appropriate treatment is:

a) Fibrinolysis

A

LINDON

How well did you know this?
1
Not at all
2
3
4
5
Perfectly
34
Q

Kate
In an adult weighing 70 kg, a bedside assessment of haemodynamic status shows a left ventricular end-diastolic diameter of 2.4 cm. This finding suggests:

a) Hypovolaemia
b) Normal
c) Hypervolaemia

A

? Normal
Can only find absolute numbers or according to BSA not weight per se

Image
https://thoracickey.com/cardiac-chambers/

PSAX End diastolic AREA:
Hypovolemia <8cm2
Normal 8-14cm 2
Hypervolemia > 14cm2
IVSd and IVSs – Interventricular septal end diastole and end systole. The normal range is 0.6-1.1 cm.
LVIDd and LVIDs – Left ventricular internal diameter end diastole and end systole. The normal range for LVIDd is 3.5-5.6 cm, and the normal range for LVIDs is 2.0-4.0 cm.
LVPWd and LVPWs – Left ventricular posterior wall end diastole and end systole. The normal range is 0.6-1.1 cm.
RVDd – Right ventricular end diastole. The normal range is 0.7-2.3 cm.
Ao Root Diam – Aortic root diameter. The normal range is 2.0-4.0 cm.
LA Diameter – Left atrium diameter. The normal range is 2.0-4.0 cm.
The IVSd and IVPWd measurements are used to determine left ventricular hypertrophy, which is the thickening of the muscle of the left ventricle. LV hypertrophy is a marker for heart disease. In general, a measurement of 1.1-1.3 cm indicates mild hypertrophy, 1.4-1.6 cm indicates moderate hypertrophy, and 1.7 cm or more indicates severe hypertrophy.

Hypovolaemia
Normal for end diastole is 3.5 to 5.6cm

How well did you know this?
1
Not at all
2
3
4
5
Perfectly
35
Q

2hyp2.1A high mixed venous oxygen saturation (SvO2) is most likely to be associated with

a. COPD
b. PE / Tamponade
c. Acute MI
d. Severe liver failure
e. Sepsis

A

d. Severe liver failure
but could also be
e. Sepsis

LIFTL:

INTERPRETATION

High SvO2
- increased O2 delivery (increased FiO2, hyperoxia, hyperbaric oxygen)
- decreased O2 demand (hypothermia, anaesthesia, neuromuscular blockade)
- high flow states: sepsis, hyperthyroidism, severe liver disease

Low SvO2
- decreased O2 delivery:
1. decreased Hb (anaemia, haemorrhage, dilution)
2. decreased SaO2 (hypoxaemia)
3. decreased Q (any form of shock, arrhythmia)

  • increased O2 demand (hyperthermia, shivering, pain, seizures)
  • Causes of High SvO2 despite evidence of End-organ Hypoxia:
    1. microvascular shunting (e.g. sepsis)
    2. histotoxic hypoxia (e.g. cyanide poisoning)
    3. abnormalities in distribution of blood flow

Anesthesia Monitoring Of Mixed Venous Saturation:
https://www.ncbi.nlm.nih.gov/books/NBK539835/

In sepsis, ScvO2 less than 70% or SvO2 lower than 65% correlate with poor prognosis.[2] In application, certain studies have shown that maintaining a goal ScvO2 greater than 70% leads to reduced mortality.[11] Therefore, ScvO2 is used to guide treatment algorithms in the Surviving Sepsis Campaign (SSC).

Studies have shown that normal to higher levels of mixed venous oxygen saturation in patients with clinically worsening sepsis do not rule out tissue hypoxia due to the inability to utilize O2.[11][7] Therefore, several studies support the conclusion that abnormally low or high ScvO2 correlates with higher mortality in patients with septic shock.

How well did you know this?
1
Not at all
2
3
4
5
Perfectly
36
Q

20.1 In the treatment of diabetic ketoacidosis, the most important initial therapeutic intervention is to

a) Electrolyte correction
b) Insulin
c) IV hydration
d) Bicarbonate

A

IV hydration

Fluid first (hartmanns or saline w k+) then insulin
BJA Developments in the management of diabetic ketoacidosis 2015

Diabetic ketoacidosis (DKA) is a medical emergency and bedside capillary ketone testing allows timely diagnosis and identification of successful treatment.

> 0.9% saline with premixed potassium chloride should be the main resuscitation fluid on the general wards and in theatre; this is because it complies with National Patient Safety Agency recommendations on the administration of potassium chloride.

> Weight-based fixed rate i.v. insulin infusion (FRIII) is now recommended rather than a variable rate i.v. insulin infusion (VRIII).

> The blood glucose must be kept above 14 mmol litre−1 with the FRIII.

> Precipitating factor(s) needs to be identified and treated. Surgery and also critical care may be indicated to manage the patient presenting with DKA.

How well did you know this?
1
Not at all
2
3
4
5
Perfectly
37
Q

22.2 The most likely diagnosis for the following electrocardiograph is
(comment that this was like a 2015A repeat - ECG below is from that paper + 2022 recalled options)

a. AF with BBB
b. sinus tachy with BBB
c. ventricular tachycardia
d. torsades

A

b. sinus tachy with BBB

The most correct answer would be Trifasicular block:
RBBB with LAD (RBBB with left anterior hemiblock) and 1st degree heart block

Barash 8E 2017:
The term bifascicular block often refers to block in the right bundle and one of the two major fascicles of the left bundle. RBBB with left anterior hemiblock is present when the ECG shows an RBBB with a left axis deviation (usually greater than −60 degrees) in the absence of an inferior myocardial infarction. Complete RBBB with right axis deviation (greater than 90 degrees) is indicative of RBBB and left posterior hemiblock in the absence of a lateral myocardial infarction or evidence of right-sided heart failure. The term trifascicular block is used to describe first-degree AV block in the presence of bifascicular block.

Is it necessary to insert a temporary pacemaker before general anesthesia for an asymptomatic patient with bifascicular or trifascicular block?
The risk for progression to complete heart block in asymptomatic patients with bifascicular block is low. Further, no clinical characteristics have been identified that accurately predict the risk of development of complete heart block. Therefore, routine PPM implantation in patients with asymptomatic bifascicular block is not recommended. Observations made in the perioperative period have suggested that development of complete heart block during general anesthesia is also rare; therefore, it is generally not recommended that patients undergo temporary pacemaker insertion before general anesthesia. However, it is advisable to have an external pacemaker available in the operating room.

How well did you know this?
1
Not at all
2
3
4
5
Perfectly
38
Q

21.1 Infection control management of patients with carbapenemase-producing Enterobacteriaceae (CPE)
infection should include all of the following EXCEPT
a) isolation
b) contact precautions
c) droplet precautions
d) screening at risk patients with rectal swab and urine mcs

A

c) droplet precautions

https://www.safetyandquality.gov.au/sites/default/files/migrated/Recommendations-for-the-control-of-Carbapenemase-producing-Enterobacteriaceae.pdf

How well did you know this?
1
Not at all
2
3
4
5
Perfectly
39
Q

23.1 A man has symptomatic carbon monoxide poisoning. His pulse oximetry (SpO2) and arterial blood gas (PaO2) would be expected to show

a. Normal SpO2, Normal PaO2
b. Normal SpO2, reduced PaO2
c. Reduced SpO2, normal PaO2
d. Reduced SpO2, reduced PaO2

A

a. Normal - Normal

A normal or high oximetry reading should be disregarded because saturation monitors cannot differentiate between carboxyhaemoglobin and oxyhaemoglobin, owing to their similar absorbances.
The blood gas PO2 will also be normal in these cases (despite the presence of tissue hypoxia).

file:///Users/newuser/Downloads/BTS%20Guideline%20for%20oxygen%20use%20in%20adults%20in%20healthcare%20and%20emergency%20settings.pdf

How well did you know this?
1
Not at all
2
3
4
5
Perfectly
40
Q

21.1 The most common cause of postoperative visual loss after spinal surgery is

a. Central retinal artery occlusion
b. Central retinal vein occlusion
c. Ischemic optic neuropathy
d. Haemorrhage
e. corneal abrasion

A

c. Ischemic optic neuropathy

Cardiac: Anterior
Spinal: Posterior
ION

How well did you know this?
1
Not at all
2
3
4
5
Perfectly
41
Q

20.2 A Jehovah’s Witness patient attends for a revision total hip replacement and is medically optimized. You consider she is high risk for the procedure but after extensive discussion agree to proceed, including agreeing that you will not give blood under any circumstances. Your decision can be justified on the basis of

a) Paternalism
b) Non maleficence
c) Autonomy
d) Beneficence

A

a) Autonomy
- Obligation to respect the decision-making capacities of persons.

Non-maleficence: Obligation to avoid causing harm
- If refused to proceed.

Paternalism: A set of attitudes and practices in which the health provider determines that a patient’s wishes or choices should not be honored.
- If transfused patient against their wishes

Beneficence: Obligation to provide benefits and to balance benefits against risks; obligation of physician to act for the benefit of the patient
- Controversial interpretation in this case. Both proceeding and refusing to do case may be acting for the benefit of the patient, depending on how you look at the scenario.

BJA: ‘MORAL balance’ decision-making in critical care
https://www.bjaed.org/article/S2058-5349(18)30145-8/fulltext

How well did you know this?
1
Not at all
2
3
4
5
Perfectly
42
Q

21.1 An 84-year-old woman with dementia presents for surgery for a breast lump. She lives in a care facility and is accompanied by the nurse manager from the facility and her son. Neither have a written legal authority to act on her behalf. Regarding consent for her surgery

a) Anaesthetic consent is implied in surgical consent
b) Son can’t consent
c) Legal guardian can’t consent
d) Not required if 2 Doctors are in agreement about the need for surgery
e) nil consent required if would be in patients interest/not against wishes

A

a) anaesthetic consent implied in surgical consent

or

e) nil consent required if would be in patients interest/ not against wishes

https://journals.sagepub.com/doi/pdf/10.1177/0310057X1003800504

The anaesthetist should give the parent or relative the same kind of information as a patient and recommend the appropriate procedure. The consent of a parent or relative is lawful authority to proceed. If the parent or relative does not wish to take this role, it may be necessary for a court or a guardianship body to appoint someone else to make the decision. This may take some time, and if the procedure is medically necessary and cannot be deferred, then reasonable treatment may be administered without consent (this is a principle of the common law and, in some jurisdictions, is also in legislation: for example, in New South Wales, the Guardianship Act 1987 (NSW) s 37; and in Victoria

How well did you know this?
1
Not at all
2
3
4
5
Perfectly
43
Q

22.2 Regarding cardiopulmonary exercise testing before major surgery, oxygen pulse is the

a. Arterial oxygen content at peak HR
b. Arterial oxygen saturation at mean HR?
c. Arterial oxygen saturation at peak HR
d. PaO2 at peak HR
e. Oxygen consumption/min divided by HR

A

e. Oxygen consumption/min divided by HR

VO2/HR: oxygen consumption divided by HR, known as the ‘oxygen pulse’ (ml beat–1)

https://www.bjaed.org/article/S2058-5349(19)30021-6/fulltext

The objective of CPET is to determine functional capacity in an individual.
Deficiencies in CPET-derived variables—specifically:
1. ventilatory anaerobic threshold (AT)
2. peak O2 consumption (VO2peak)
3. ventilatory efficiency for carbon dioxide (VE/VCO2)
—are associated with poor postoperative outcomes (mortality, morbidity, admission to intensive care, and length of hospital stay) after intra-abdominal surgery.

  1. Does the oxygen pulse increase with exercise?
    The oxygen pulse is the VO2 divided by HR, and represents the product of the stroke volume and the arterial-venous oxygen difference. It can be seen in panel 2 and can be viewed as a surrogate for stroke volume, and as such should increase at the start of exercise before slowly reaching a plateau at its highest predicted value.
How well did you know this?
1
Not at all
2
3
4
5
Perfectly
44
Q

23.1 According to the Fourth Consensus Guidelines for the Management of Post-operative Nausea and Vomiting (PONV) published in 2020, multimodal PONV prophylaxis should be implemented in adult patients

a. For everyone
b. 1 or more RF
c. 2 or more RF
d. 3 or more RF
e. 4 or more RF

A

b) 1 or more RF

How well did you know this?
1
Not at all
2
3
4
5
Perfectly
45
Q

22.2 The prevention of microbial contamination of living tissues or sterile materials is known as

a. disinfection
b. antisepsis
c. decontamination
d. asepsis
e. sterilisation

A

d. asepsis

Asepsis: the prevention of microbial contamination of living tissues or sterile materials.
Disinfection: the inactivation of non-sporing organisms using either thermal or chemical means.
Sterilisation: complete destruction of all micro-organisms, including spores.

https://www.anzca.edu.au/getattachment/e4e601e6-d344-42ce-9849-7ae9bfa19f15/PG28(A)-Guideline-on-infection-control-in-anaesthesia

How well did you know this?
1
Not at all
2
3
4
5
Perfectly
46
Q

22.1 The risk of a perioperative respiratory adverse event in a child is least likely to be increased by

a. Asthma
b. Infection 3 weeks ago
c. History of eczema
d. Passive smoking

A

History of eczema

APRICOT study

How well did you know this?
1
Not at all
2
3
4
5
Perfectly
47
Q

22.2 A patient with an acute subarachnoid haemorrhage arrives in the emergency department. Her Glasgow Coma Scale score is 10 and she has no motor deficit. A CT brain shows diffuse subarachnoid haemorrhage with no localised areas of blood > 1 mm thick, and no intracerebral nor intraventricular blood. Her World Federation of Neurosurgical Societies (WFNS) grade of subarachnoid haemorrhage is

a) 1
b) 2
c) 3
d) 4
e) 5

A

4

GCS 7-12

How well did you know this?
1
Not at all
2
3
4
5
Perfectly
48
Q

22.1 A risk factor for postoperative nausea and vomiting in adults is age less than

a. 20
b. 30
c. 40
d. 50
e. 60

A

50

4th consensus guidelines for management of PONV

How well did you know this?
1
Not at all
2
3
4
5
Perfectly
49
Q

An awake patient in the post-anaesthesia care unit complains of breathlessness. The FiO2 through the patient’s rebreather mask is 40%. An arterial blood gas taken at the time shows (ABG shown). The alveolar-arterial gradient (in mmHg) is approximately

Blood gas shows:
PaO2 135
PaCO2 48
SpO2 100%

The A-a gradient is:
A. 5
B. 30
C. 60
D. 90
E. 110

A

D 90

A-a = PAO2 - PaO2

Alveolar air equation gives PAO2

PAO2 = PiO2 - PaCO2 / R
PAO2 = 0.4 x (760 - 47) - 48 / 0.8

so, as PaO2 given as 135
A-a = 228 - 135 = 93

How well did you know this?
1
Not at all
2
3
4
5
Perfectly
50
Q

21.2 You have been asked to provide general anaesthesia for a complex thoracic endovascular aortic aneurysm repair. After the placement of a lumbar drain the recommended safe time before the administration of intravenous heparin is

a) 1 hour
b) 4 hours
c) 6 hours
d) 12 hours

A

1 hour

ASRA: 1 hour

Although the occurrence of a bloody or difficult neuraxial needle placement may increase risk, there are no data to support mandatory cancellation of a case. Direct communication with the surgeon and a specific risk-benefit decision about proceeding in each case are warranted.

Currently, insufficient data and experience are available to determine if the risk of neuraxial haematoma is increased when combining neuraxial techniques with the full anticoagulation of cardiac surgery. We suggest postoperative monitoring of neurologic function and selection of neuraxial solutions that minimise sensory and motor block to facilitate detection of new/progressive neurodeficits.

NYSORA:
Administration of intravenous heparin intraoperatively should be delayed for at least 1 hour after epidural placement; a delay before administration of subcutaneous heparin is not required. In cases of full heparinization for CPB, additional precautions include delaying surgery for 24 hours in the event of a traumatic tap, tightly controlling the heparin effect and reversal, and removing catheters when normal coagulation is restored.

How well did you know this?
1
Not at all
2
3
4
5
Perfectly
51
Q

22.1 The dose of hydrocortisone that has equivalent glucocorticoid effect to 8 mg dexamethasone is

a) 12mg
b) 25mg
c) 50mg
d) 100mg
e) 200mg

A

200mg Hydrocortisone or 25mg Prednisolone

Conversion
Prednisone 1mg =
Hydrocortisone 4mg =
Dexamethasone 0.15mg =
Triamcinolone 0.8mg =
Methylprednisolone 0.8mg =
Betamethasone 0.15mg =

(https://litfl.com/corticosteroids-overview/)

How well did you know this?
1
Not at all
2
3
4
5
Perfectly
52
Q

20.2 The most common type of perioperative stroke is

a) Hypoxic
b) Thrombotic
c) Embolic
d) Hypotensive
e) Haemorrhagic

A

c) Embolic

Blue Book 2017
Perioperative Stroke

Epidemiology
A perioperative stroke is defined as one that occurs either intra-operatively or in the post-operative period within 30 days70. Perioperative strokes are associated with an increased length of stay and a six-fold increased mortality. Any combination of surgery and anaesthesia is associated with an increased risk of stroke irrespective of the type of surgery. This may relate to coagulation changes

The most common type of perioperative stroke is ischaemic stroke of embolic origin (heart or aorta). Hypotension is rarely the cause of perioperative stroke. Haemorrhagic stroke is uncommon which probably reflects the fact that severe hypertension during anaesthesia is a rare event, and anticoagulants have typically been withheld.

The risk of perioperative stroke varies depending on the type of the surgery and patients’ risk factors.

Procedural risk
Urgent surgery is associated with an increased risk of stroke when compared to elective surger.

Cardiac, vascular and brain surgeries are defined as “high-risk” as these have an increased risk of stroke when compared to other types of surgery. Valvular and aortic repair surgeries have a stroke risk as high as 8 to 10 per cent.

Perioperative strokes in non-high-risk surgery are relatively rare and are estimated to have an incidence of about 1/1000 cases80.

Patients’ risk factors
>Age
>history of previous stroke or transient ischaemic attack
>renal failure
>atrial fibrillation
>history of cardiovascular diseases
are identified risk factors for perioperative stroke.

Atrial fibrillation is associated with a two-fold increase in the risk of death and stroke after carotid endarterectomy.

How well did you know this?
1
Not at all
2
3
4
5
Perfectly
53
Q

21.1 In the morbidly obese the induction dose of propofol should be calculated based on

a. Lean body weight
b. Total body weight
c. Ideal body weight
d. Ideal body weight + 70%

A

Lean Body Weight

For infusion: Adjusted body weight
NDMB: Lean Body weight
Sux: Total body weight

Source: SOBA UK

How well did you know this?
1
Not at all
2
3
4
5
Perfectly
54
Q

A 50-year-old woman has had a headache for the last month which is relieved by lying flat. She has had no medical procedure to her spine such as epidural, spinal or lumbar puncture. Her brain magnetic resonance imaging (MRI) scan shows diffuse meningeal enhancement and brain sagging. Her neurologist suspects spontaneous intracranial hypotension and asks you to do an epidural blood patch. No spinal imaging has been performed to confirm a cerebrospinal fluid (CSF) leak. You should

A do LP to measure pressure if low do lumbar patch
B do blood patch at lumbar level with no further investigation
C do spine imaging if CSF leak present do blood patch at level
D do spine imaging if CSF leak present do lumbar blood patch
E refuse to do blood patch

A

B do blood patch at lumbar level with no further investigation

How well did you know this?
1
Not at all
2
3
4
5
Perfectly
55
Q

19.2 An 80-year-old woman is admitted to hospital with respiratory failure. Her arterial blood gas on oxygen 4 litres per minute via a Hudson mask is as follows: (ABG shown) Which of the following most accurately describes this blood gas result?

pH 7.2, pO2 91, pCO2 84, BE 16, HCO3- 43, Na 145

a) Metabolic alkalosis, acute resp acidosis + normal AG
b) Metabolic alkalosis resp acidaemia + abnormal AG
c) Mixed acidaemia
d) Respiratory Acidosis with incomplete compensation
e) Compensated Respiratory acidosis

A

d) Respiratory Acidosis with incomplete compensation

Uncertain of this answer, not enough info to calculate anion gap

pH 7.2 = acidaemia
pCO2 84 = respiratory acidosis
HCO3 43 = metabolic alkalosis as compensation
BE 16 = metabolic alkalosis

Boston rules:
Chronic fully compensated Respiratory acidosis
Expected compensation is 3-4 mmol/L rise for every 10mmHg rise in PCO2.
Expected metabolic compensation therefore is
HCO3 = 24 + 4 x ((84-40)/10)
= 24 + 4x (44/10)
= 24 + 4 x (4.4)
= 24 + 17.6
= 41.6

Metabolic acidosis
PaCO2 should be 1.5 x HCO3 + 8
= 72.5

Rules (from K.Brandis Acid-base rules anaesthesia mcq):
- 1 for 10 (acute resp acidosis), 4 for 10 (chronic resp acidosis)
- 2 for 10 (acute resp alkalosis), 5 for 10 (chronic resp alkalosis)
- 1.5xHCO + 8 = expected pCO2 in a metabolic acidosis
- 0.7xHCO3 + 22 = expected pCO2 in a metabolic alkalosis

https://www.anaesthesiamcq.com/AcidBaseBook/ab9_3.php

How well did you know this?
1
Not at all
2
3
4
5
Perfectly
56
Q

20.1 ECG calibration, 10mm on Y axis is equal to:

a. 0.2 sec
b. 0.4sec
c. 1sec
d. 0.1mV
e. 1mV

A

b) 1mV

How well did you know this?
1
Not at all
2
3
4
5
Perfectly
57
Q

22.1 A 68-year-old woman presents with a loud systolic murmur in the anaesthesia room before total
hip joint arthroplasty. A transthoracic echocardiogram is performed (image provided) and shows

a. AS
b. LVOT
c. MR

A

MR

How well did you know this?
1
Not at all
2
3
4
5
Perfectly
58
Q

22.1 The most reliable clinical indicator of opioid-induced ventilatory impairment (OIVI) is decreased

a. Sedation
b. Respiratory rate

A

SS /GCS

Repeat APMSE

How well did you know this?
1
Not at all
2
3
4
5
Perfectly
59
Q

21.1, 23.1 In patients without other co-morbidities, bariatric weight loss surgery is indicated when the body mass index (kg/m2) is greater than

A

a. 35

Major updates (2022) to 1991 National Institutes of Health guidelines for bariatric surgery

Metabolic and bariatric surgery (MBS) is recommended for individuals with a body mass index (BMI) 35 kg/m2 , regardless of presence, absence, or severity of co-morbidities.

MBS should be considered for individuals with metabolic disease and BMI of 30-34.9 kg/m2

BMI thresholds should be adjusted in the Asian population such that a BMI 25 kg/m2 suggests clinical obesity, and individuals with BMI 27.5 kg/m2 should be offered MBS.

Long-term results of MBS consistently demonstrate safety and efficacy.

Appropriately selected children and adolescents should be considered for MBS.

https://www.soard.org/article/S1550-7289(22)00641-4/fulltext#:~:text=The%201991%20NIH%20Consensus%20Statement,surgery%20that%20is%20applied%20universally

Contraindications:
- Inflammatory disease of GI tract (ulcers, oesophagitis, Crohn’s)
- Upper GI bleeding
- Portal Htn
- Liver Cirrhosis
- Chronic Pancreatitis
- Laparascopic surgery may be technically difficult in patients weighing >180kg and may be considered a relative contraindication

How well did you know this?
1
Not at all
2
3
4
5
Perfectly
60
Q

21.2 Of the following, the lifestyle modification that is least effective in reducing essential
hypertension is

a) Stopping caffeine
b) Low salt diet
c) High potassium diet
d) Exercise
e) Alcohol cessation

A

a) Stopping caffeine

Eat a well-balanced diet that’s low in salt
Limit alcohol
Enjoy regular physical activity
Manage stress
Maintain a healthy weight
Quit smoking

Foods that are rich in potassium are important in managing high blood pressure (HBP or hypertension) because potassium lessens the effects of sodium. The more potassium you eat, the more sodium you lose through urine. Potassium also helps to ease tension in your blood vessel walls, which helps further lower blood pressure.

Source AHA

How well did you know this?
1
Not at all
2
3
4
5
Perfectly
61
Q

23.1 Diagnostic criteria for adult systemic inflammatory response syndrome include all of
the following EXCEPT

a. Leukopenia
b. Hypothermia
c. Tachycardia
d. Tachypnoea
e. Hypotension

A

e. Hypotension

https://www.safetyandquality.gov.au/sites/default/files/2022-06/sepsis_clinical_care_standard_2022.pdf

How well did you know this?
1
Not at all
2
3
4
5
Perfectly
62
Q

21.2, 22.2, 23.2 A 25-year-old woman is administered two doses of aprepitant for postoperative nausea and vomiting after a sleeve gastrectomy. She normally takes the oral contraceptive pill. You should advise her to use alternative contraception for the next

a) 3 days
b) 7 days
c) 14 days
d) 28 days

A

28 days

Aprepitant PI:
“Alternative or “back-up” measures of contraception should be used during treatment with this medicine and for one month following the last dose of this medicine.”

Pharmacokinetics:
- aprepitant is a CYP3A4 inhibitor
- caution is also advised with warfarin and phenytoin use

How well did you know this?
1
Not at all
2
3
4
5
Perfectly
63
Q

21.1 The modified Aldrete scoring system uses all of the following EXCEPT

a) BP
b) Pain score
c) Resp rate
d) sedation level

A

pain score

How well did you know this?
1
Not at all
2
3
4
5
Perfectly
64
Q

22.1 Red man syndrome as a consequence of vancomycin administration is caused by

a. Type II hypersensitivity reaction
b. IgE sensitivity
c. Vasodilation of vessels
d. Mast cell degranulation

A

Mast cell degranulation - anaphylactoid reaction

How well did you know this?
1
Not at all
2
3
4
5
Perfectly
65
Q

22.1 A 57-year-old female smoker presents for a laparotomy with the following pulmonary function tests
(normal FEV1 FVC, low RV and FRC only, normal DLCO)
They are consistent with a diagnosis of

a. Obesity
b. PE
c. Pulmonary fibrosis
d. COPD

A

a. Obesity

Obesity and pulmonary function testing
https://www.jacionline.org/article/S0091-6749(05)00164-8/fulltext

  • Full pulmonary function tests are often necessary to better characterize the spirometric abnormalities seen in the obese patient
  • The most sensitive indicator of obesity is a low expiratory reserve volume (ERV) and functional residual capacity
  • Restriction is seen in more severe obesity, with reductions in TLC and FVC.
  • However, residual volume is often preserved because of the relative high closing volume in relation to ERV.
How well did you know this?
1
Not at all
2
3
4
5
Perfectly
66
Q

20.2 You are seeing a 48 year-old woman in your pre-operative clinic for assessment for laparoscopic sleeve gastrectomy. Her co-morbidities include obesity (BMI is 65 kg/m2), hypertension, type 2 diabetes mellitus and polycystic ovary syndrome. Her neck circumference is 38 cm. Her husband states that she snores loudly, but he has never observed her having any apnoeic episodes and she reports no excessive tiredness during the day. Her score using the STOP-BANG questionnaire is

a. 3
b. 4
c. 5
d. 6
e. 7

A

a. 3 (snoring, BMI, Htn)

Snoring loudly
Tiredness during day time
Observed Apnoea
Pressure: Htn

BMI > 35
Age > 50
Neck circumference >40cm (43cms male)
Gender: Male

How well did you know this?
1
Not at all
2
3
4
5
Perfectly
67
Q

23.1 The following is a chest X-ray from a patient complaining of dyspnoea after thoracic surgery. The diagnosis is

(not the image from the exam)

A. Dextracardia
B. Cardiac hernation
C. LLL collapse
D. Tension Pneumohorax

A

B. Cardiac hernation

https://www.ahajournals.org/doi/10.1161/CIRCULATIONAHA.109.896829

How well did you know this?
1
Not at all
2
3
4
5
Perfectly
68
Q

22.2 All of the following conditions are associated with acromegaly EXCEPT

a) cardiac arrhythmias
b) cardiac failure
c) OSA
d) aortic dilation

A

d) aortic dilation

Osteoarthritis
nerve compression syndrome due to bony overgrowth, and carpal tunnel syndrome
Hypertension
Diabetes mellitus
Cardiomyopathy/HF
Colorectal cancer
Sleep Apnea
Thyroid nodules and thyroid cancer
Hypogonadism
Compression of the optic chiasm

Source: BJA

How well did you know this?
1
Not at all
2
3
4
5
Perfectly
69
Q

20.2 You are anaesthetising a 35 year old woman undergoing a laparoscopic appendicectomy. She uses a levonorgestrel-secreting intrauterine device (MirenaTM) for contraception and you have used sugammadex for reversal of neuromuscular blockade at the end of the procedure. Your post-operative
advice to her regarding contraception should state that

a. Barrier protection for a week
b. Barrier protection until the next period.
c. The mirena is sufficient
d. OCP for a week
e. OCP until next period

A

a. Barrier protection for a week

In the case of non-oral hormonal contraceptives, the patient must use an additional non hormonal contraceptive method for the next 7 days

How well did you know this?
1
Not at all
2
3
4
5
Perfectly
70
Q

22.2 For a 70-year-old patient on rivaroxaban with normal renal function a major guideline recommends proceeding with hip fracture surgery after two half-lives of the drug. This equates to

a. 12 hours
b. 24 hours
c. 48 hours
d. 72 hours
e.

A

b. 24 hours

ASA guidelines

-If creatinine clearance >/=30 ml.min-1 (Cockcroft-Gault), proceed with surgery after two half lives (24 h) since the last dose, under general anaesthesia (or spinal anaesthesia if indicated)
- If creatinine clearance < 30 ml.min-1, proceed with surgery after four half lives (48 h) since the last dose, under general anaesthesia (or spinal anaesthesia if indicated)

How well did you know this?
1
Not at all
2
3
4
5
Perfectly
71
Q

21.2 A respiratory effect of high flow nasal oxygen therapy is

a) Increased deadspace
b) Reduced MV
c) Increased work of breathing
d) Reduced RR

A

d) Reduced RR

BJA: HFNP oxygen therapy
https://www.bjanaesthesia.org/article/S0007-0912(17)53999-9/fulltext
- reduced RR
- increased MV
- reduced WOB, reduced Vd, reduced AWR
- provides CPAP 3-7 cmH20 (mouth closed)

How well did you know this?
1
Not at all
2
3
4
5
Perfectly
72
Q

20.2 A 56 year old patient presents with exertional syncope. The most likely diagnosis is

a) HOCM
b) Long QT
c) CCF
d) Myocardial ischaemia

A

HOCM if these remembered options are correct

Alternative is Aortic Stenosis which is more common than HOCM in this age group

How well did you know this?
1
Not at all
2
3
4
5
Perfectly
73
Q

21.1 Of the following classes of medication for diabetes mellitus, the most likely to cause hypoglycaemia in the fasted patient are the

A. Biguanides (metformin)
B. Sulphonylureas (gliclazide)
C. Acarbose
D. SGLT2 inhibitors (empaglaflozin)
E. DPP4 inhibitors (sitagliptin)

A

Absolute most = Insulin, but probably not an option.

Sulphonylureas most likely

How well did you know this?
1
Not at all
2
3
4
5
Perfectly
74
Q

20.1 Perioperative overheating is most likely to cause worsening of symptoms of

A) Duchenne Muscular dystrophy
B) Myasthenia gravis
C) Multiple sclerosis
D) Myotonica dystrophia
E) Eaton Lambert syndrome

A

Answer: c) MS

CEACCP 2012 Neuromuscular disorders and anaesthesia. Part 2: specific neuromuscular disorders

Multiple sclerosis
This is the most frequently occurring demyelinating neuromuscular disorder. It is a chronic relapsing condition characterized by the formation of plaques within the brain and spinal cord. These plaques cause demyelination around the axons, resulting in weakness and spasticity as well as sensory dysfunction.
Anaesthetic considerations. Local anaesthetics may exacerbate symptoms due to the increased sensitivity of demyelinated axons to local anaesthetic toxicity.
Non-depolarizing neuromuscular blocking agents may be used in normal doses. Caution should be exercised when using depolar- izing neuromuscular blocking agents if the patient is debilitated. Temperature maintenance is important as symptoms can deteriorate with an increase in temperature, as demyelinated axons are also more sensitive to heat.

BJA: Perioperative management of myasthenia gravis (2021 - written after this MCQ):

Several factors, many associated with surgery and anaesthesia, may exacerbate myasthenia or lead to a myasthenic crisis, a life-threatening condition in which severe respiratory muscle insufficiency leads to respiratory failure.
Crises are most commonly precipitated by infection. Other precipitants include surgery, residual neuromuscular block, pain, many drugs, hypo- and hyperthermia, reduction or withdrawal of treatment, pregnancy, stress and sleep deprivation.

How well did you know this?
1
Not at all
2
3
4
5
Perfectly
75
Q

20.2 A patient presents with a serum sodium of 110mmol/L. A feature NOT consistent with a diagnosis of syndrome of inappropriate antidiuretic hormone (SIADH) is

a. urinary sodium >40
b. Euvolemia
c. Increased cortisol
d. Urine osmolarity <100
e. Serum Na <145

A

d. Urine osmolarity <100

DIAGNOSTIC CRITERIA
>hypotonic hyponatraemia
>urine osmolality > plasma osmolality (<275mOsm/kg) (i.e. concentrated urine despite hypotonic blood)
>urinary Na+ > 20mmol/L
>normal renal, hepatic, cardiac, pituitary, adrenal and thyroid function
>euvolaemia (absence of hypotension, hypovolaemia, and oedema)
correction by water restriction

CAUSES (MAD CHOP)

Major Surgery
>abdominal
>thoracic
>transsphenoidal pituitary surgery (6-7 days post op)

ADH production by tumours (Ectopic)
>small cell bronchogenic carcinoma
>adenocarcinoma of pancreas/duodenum
>leukaemia
>lymphoma
>thymoma

Drugs
>antidepressants (e.g. SSRI, TCAs, MAOIs)
>psychotropics (e.g. haloperidol, chlorpromazine), carbamazepine, Na+ valproate)
>anaesthetic drugs (barbiturates, inhalational agents, oxytocin, opioids)
>ADH analogues (vasopressin, DDAVP)
>chemotherapy (e.g.Vinca alkaloids, Melphalan, Methotrexate and cyclophosphamide)
>others (e.g. NSAIDs, amiodarone, ciprofloxacin, morphine, MDMA, proton pump inhibitors)

CNS Disorders
>cerebral trauma
>brain tumour (primary or metastases)
>meningitis/encephalitis
>brain abscess
>SAH
>acute intermittent porphyria
>SLE

Hormone deficiency
>hypothyroidism
>adrenal insufficiency

Others
>Guillain-Barre Syndrome
>HIV infection (early symptomatic or AIDS)
>hereditary SIADH
>giant cell arteritis
>idiopathic (occult small cell or olfactory neuroblastoma)

Pulmonary Disorders
>pneumonia (viral, fungal, bacterial)
>TB
>lung abscess

MANAGEMENT
1. see hyponatraemia
2. fluid restrict
3. incremental increase in Na+ if indicated to avoid central pontine myelinolysis
4. medications to decrease ADH secretion
>Demeclocycline
>Tolvaptan / Conivaptan

How well did you know this?
1
Not at all
2
3
4
5
Perfectly
76
Q

20.2 The composition of Plasma-Lyte 148 (in mmol/l) includes

a Na 140 Mg 1.0 K 5.0 acetate 27 lactate 0
b Na 140 Mg 1.5 K 5.0 acetate 0 lactate 27
c Na 140 Mg 1.0 K 4.0 acetate 24 lactate 0
d Na 140 Mg 1.0 K 4.0 acetate 0 lactate 24
e Na 140 Mg 1.5 K 5.0 acetate 27 lactate 0

A

e Na 140 Mg 1.5 K 5.0 acetate 27 lactate 0

How well did you know this?
1
Not at all
2
3
4
5
Perfectly
77
Q

21.2 Suxamethonium causes a sustained contraction of the extraocular muscles for up to

a) 2 minutes
b) 3 minutes
c) 5 minutes
d) 10 minutes
e) 20 minutes

A

d) 10 minutes
- best answer; one of those shit questions that depends on your source.

Morgan & Mikhail’s (chapter 36: anaesthesia for ophthalmic surgery):
“ Succinylcholine increases IOP by 5-10mmHg for 5-10 minutes”.
- due to prolonged contracture of the EOM

BARASH:
Succinylcholine increases IOP 7 to 10 mmHg reaching a peak pressure 1 to 2 minutes after IV administration and returns to the baseline in 5 to 7 minutes. This increase may be attenuated by pretreatment with anesthetics, although none completely eliminates the increase in IOP. In the presence of a lacerated globe, this increase in IOP may increase the extrusion of intraocular contents although greater increases in IOP may occur during crying and coughing.

Yao & Artusio’s:
- also quotes same information: increases IOP 7 to 10mmHg, returning to baseline in 5 - 7 minutes.

Stoelting’s:
Intraoccular pressure peaks at 2-4 minutes after administration and returns to normal by 6 minutes

How well did you know this?
1
Not at all
2
3
4
5
Perfectly
78
Q

22.1 Propofol infusion syndrome is characterised by all of the following EXCEPT

a. Splenomegaly
b. ST elevation
c. Hepatomegaly
d. Rhabdomyolysis
e. Metabolic acidosis

A

a. Splenomegaly

Associated with high doses >4mg/kg/hr and prolonged use (>48hrs)
Safe doses of propofol infusion for sedation in ICU are considered to be 1-4mg/kg/hr
-> fatal Cases pf PRIS have been reported after infusion doses as low as 1.9-2.6mg/kg/hr

Risk factors:
i. Young age
ii. Critical illness
iii. High fat and low Carbohydrate intake
iv. Inborn errors of mitochondrial fatty acid oxidation
v. Catecholamine infusion/ High catecholamine and glucocorticoid levels
vi. Steroid therapy
vii. Severe head injuries

Characteristics:
i. Bradycardia
ii. Severe metabolic acidosis
iii. Cardiovascular collapse
iv. Rhabdomyolysis
v. Hyperlipidaemia
vi. Renal failure
vii. Hepatomegaly

Management:
- Routine monitoring of CK and triglycerides should be performed for the at risk population
○ Daily CK and triglyceridees after 48hrs of propofol infusion
○ Increasing CK in the absence of other pathology triggers suspiscion of PRIS
- Propofol immediately stopped and alternative (midazolam and alfentanil) are used
- PRIS is difficult to treat once it occurs
- CVS support provided as needed
- Renal replacement therapy may be required to treat lactic acidosis, clear propofol and its metabolites from the patient rapidly
- Catecholamine resistant shock has been reported
- Pacing has been used with limited success
ECMO has been reported and successfully used in the CVS support of PRIS

How well did you know this?
1
Not at all
2
3
4
5
Perfectly
79
Q

21.1 A respiratory effect of high flow nasal oxygen therapy is

A. Reduced RR
B. Reduced MV
C. Increased work of breathing

A

A. Reduced RR

BJA HFNOT

It has been demonstrated that patients with acute hypoxaemic respiratory failure experience improved comfort and tolerance with HFNOT compared with humidified oxygen via a facemask, and traditional non-invasive ventilation masks. Subjective feelings of dyspnoea AND RESPRIATORY RATES are REDUCED as is airway dryness.

How well did you know this?
1
Not at all
2
3
4
5
Perfectly
80
Q

22.2 Based on this ECG tracing, the mode in which this pacemaker is operating is

a) VAI with intermittent failure to capture
b) AAI with intermittent failure to sense
c) DDD
d) VVI with intermittent failure to capture
e) VVI with intermittent failure to sense

A

e) VVI with intermittent failure to sense

How well did you know this?
1
Not at all
2
3
4
5
Perfectly
81
Q

22.1 A 74-year-old man presents for a femoral popliteal artery bypass procedure for peripheral limb ischaemia. Regarding its role in modifying his perioperative cardiovascular risk, clonidine

a. Increased stroke
b. No change in complications
c. Increased death
d. Increased non fatal MI
e. Increased risk of non fatal cardiac arrest

A

e. Increased risk of non fatal cardiac arrest

POISE II
* clonidine 200mcg per day - did not reduce the rate of composite outcome of death or nonfatal MI - but it increased the risk of clinically important hypotension and nonfatal cardiac arrest
* aspirin initiation or continuation – no significant effect on rate of composite of death or non fatal MI but increased risk of major bleeding

How well did you know this?
1
Not at all
2
3
4
5
Perfectly
82
Q

20.1 Compared to a normothermic patient, a patient with mild intraoperative hypothermia (35.0o C) will have

a. increased bleeding and normal aptt and inr
b. Increased bleeding and decreased inr
c. Increased bleeding and decreased aptt
d. Decreased bleeding

A

a. increased bleeding and normal aptt and inr

Bleeding because cold = we know this

Haemtology analyzer in labs warms blood to 37.2 degrees (fixes hypothermia on sample)

How well did you know this?
1
Not at all
2
3
4
5
Perfectly
83
Q

22.2 The medication most strongly associated with an acute primary hypotensive reaction following transfusion of blood products is
a. aspirin
b. celecoxib
c. hydralazine
d. metoprolol
e. labetalol
f. perindopril

A

f. perindopril

Hypotensive transfusion reactions, which account for almost 3% of all transfusion reactions, are associated with patients treated with angiotensin-converting enzyme inhibitors. The current hypothesis suggests that they are caused by bradykinin-induced vasodilation in the absence of allergic, hemolytic, or septic mechanisms. The hypotension observed frequently is unresponsive to conventional therapy with catecholamines. The suggested intraoperative management includes cessation of transfusion and washing red blood cells before blood replacement.

Hypotensive reactions to transfusion may not always be recognized. To prevent these reactions, clinicians have several options: they may discontinue the ACE inhibitor (elective transfusion), not use a leukoreduction filter (if the patient has no absolute requirement for leukoreduced blood components), use washed cellular components, or use components that have undergone leukoreduction at the collection facility or the hospital blood bank before transfusion (since bradykinin is degraded during storage).

How well did you know this?
1
Not at all
2
3
4
5
Perfectly
84
Q

20.1 Cardiovascular effects of hyperthyroidism include

a) decreased diastolic relaxation
b) decreased SVR
c) decreased PVR
d) increased diastolic BP

A

Decreased SVR
- increased CO, increased SBP and decreased DBP with widened PP

Up to Date
Cardiovascular - Patients with hyperthyroidism have an increase in cardiac output, due both to increased peripheral oxygen needs and increased cardiac contractility. Heart rate is increased, pulse pressure is widened, and peripheral vascular resistance is decreased

How well did you know this?
1
Not at all
2
3
4
5
Perfectly
85
Q

22.1 A 74-year old man complains of chest pain. An electrocardiograph is performed and displayed here. The occluded coronary artery could be the

a) RCA or LCx
b) RCA
c) LAD

A

RCA or LCx

https://litfl.com/mi-localization-ecg-library/

How well did you know this?
1
Not at all
2
3
4
5
Perfectly
86
Q

22.1 A test for a condition which has a prevalence of 1 in 1000 has a sensitivity of 100% and a specificity of 90%.
The probability of a patient who receives a positive result actually having the condition is

a. 1%
b. 10%
c. 50%
d. 100%

A

a. 1%

i.e. what is the positive predictive value (PPV) for this test

PPV= TP/ TP +FP
Negative Predictive Value = TN / TN + FN

Prevalence of 1/1000
Sensitivity of 100%
Specificity of 90%

Of patients that are disease positive in population of 1000
TP = 1
FN = 0
-> 100% sensitivity

Of patients that are disease negative in population of 1000
FP = 99
TN = 900
-> 90% Specificity

PPV= 1/ 1 + 99
= 1/100
=1%

NPV= 900/ 900 + 0
= 1/1
= 100%

How well did you know this?
1
Not at all
2
3
4
5
Perfectly
87
Q

23.1 The glucagon-like peptide-1 receptor (GLP-1) agonist semaglutide is associated with

A. delayed gastric emptying
B. hypoglycaemia
C. hyperlactataemia

A

a) delayed gastric emptying

How well did you know this?
1
Not at all
2
3
4
5
Perfectly
88
Q

22.1 A risk factor for the development of torsade de pointes is

a. hyperkalaemia
b. hypermagnasaemia
c. tachycardia
d. Female

A

d. Female

How well did you know this?
1
Not at all
2
3
4
5
Perfectly
89
Q

23.1 A 24-year-old man has been brought into the emergency department with a traumatic fracture of the femur. His observations are: heart rate 90 beats per minute; blood
pressure 120/80 mmHg; respiratory rate 25 breaths per minute. A peripheral VENOUS blood gas sample shows a pH of 7.29. The arterial blood pH can be estimated to be

A. 7.29
B. 7.32
C. 7.35
D. 7.4

A

B. 7.32

https://emj.bmj.com/content/18/5/340

The values of pH on arterial and venous samples were highly correlated (r=0.92) with an average difference between the samples of −0.4 units. There was also a high level of agreement between the methods with the 95% limits of agreement being −0.11 to +0.04 units.

https://litfl.com/vbg-versus-abg/

pH
- Good correlation
- pooled mean difference: +0.035 pH units

How well did you know this?
1
Not at all
2
3
4
5
Perfectly
90
Q

23.1 During standard diagnostic nocturnal polysomnography for investigation of obstructive sleep apnoea, apnoea is defined as cessation of airflow for

A. 10 sec
B. 20 sec
C. 30 sec
D. 10 sec with 3% desat
E. 20 sec with 3 % desat

A

A

Apnea is defined as the cessation of airflow for ten or more seconds.

Hypopnea is defined as a recognizable, transient reduction, but not a complete cessation of, breathing for ten or more seconds.

Hypopnea requires a 4% fall in SpO2

https://www.ncbi.nlm.nih.gov/books/NBK441909/#:~:text=Obstructive%20Sleep%20Apnea%20(OSA)%2C,for%20ten%20or%20more%20seconds.

How well did you know this?
1
Not at all
2
3
4
5
Perfectly
91
Q

A 35-year-old male, three days post laparoscopic sleeve gastrectomy has ongoing nausea and vomiting. His arterial blood gas measurement is as follows: (ABG shown) The best initial therapeutic option would be

Blood gas given:
hypokalaemia
hypochloraemia
alkalosis
normal lactate

a Laparoscopy
b IV fluids and KCL
c 4% albumin
d HCl infusion
e Acetazolamide

A

b IV fluids and KCL

UTD Stricture post Lap Sleeve Gastrectomy management

Although sleeve strictures have been reported in 0.26 to 4 percent of LSG operations, <1 percent result in symptoms that require endoscopic or surgical intervention

A stricture can manifest acutely, early after surgery, or more chronically.

Although strictures can occur anywhere along the long staple line, they are most often located at the level of the incisura angularis for anatomic reasons.

The etiologies of post-LSG strictures are either mechanical or functional. Mechanical strictures usually derive from the use of small bougies, stapling too close to the bougie (especially at the incisura angularis), twisting of the staple line creating a “spiral” sleeve, or aggressive imbrication of the staple line.

Functional stenoses derive from edema or hematomas at the staple line. As a result, functional stenoses are transient, which present immediately following LSG and resolve spontaneously with expectant treatment.

Patients who present with obstructive symptoms during the early postoperative period should be resuscitated with hydration and antiemetic medications and studied with an upper gastrointestinal (UGI) series.

Stable patients with a stricture can be observed to allow postsurgical mucosal edema to resolve, typically in 24 to 48 hours. Patients who cannot handle their own secretions require nasogastric tube decompression, preferably placed under fluoroscopic guidance.

Patients with an acute stricture who do not respond to conservative management require early surgical reintervention. Laparoscopy could demonstrate kinking of the gastric tube, a tight suture, or a compressing hematoma.

●Endoscopy is a good initial treatment for short-segment strictures, most of which can be dilated with balloons. Multiple treatments in four- to six-week intervals are sometimes needed to treat the stricture and improve patient symptoms. Stents have also been tried but are not effective for post-LSG strictures.

●Laparoscopic seromyotomy is a treatment option for long-segment strictures . In a small retrospective study, patients treated with laparoscopic seromyotomy had good symptomatic relief.

●Conversion to an RYGB is the last option for patients with a refractory stricture who have failed all other treatments.

How well did you know this?
1
Not at all
2
3
4
5
Perfectly
92
Q

20.1 IgE-related penicillin anaphylaxis crossover rate with cephazolin

a. 0.1%
b. 1%
c. 5%
d. 10%

A

1%

BJA ED

How well did you know this?
1
Not at all
2
3
4
5
Perfectly
93
Q

23.1 According to the Australian and New Zealand Anaesthetic Allergy Group (ANZAAG) guidelines for the investigation of a suspected anaphylactic reaction, serum tryptase should be measured at

a. 0, 4, 12
b. 0, 2, 4, 24
c. 0, 1, 4, 24
d. 0, 4 , 6, 24
e. 1, 6, 24

A

c) 0, 1, 4, 24

Serum tryptase levels are recommended to be collected as soon as possible after the onset of symptoms and then at 1 hour, 4 hours and after 24 hours.

https://www.anzca.edu.au/resources/professional-documents/endorsed-guidelines/anaphylaxis-guideline-2022.pdf

How well did you know this?
1
Not at all
2
3
4
5
Perfectly
94
Q

22.2 For a skewed distribution of data the best measure of dispersion of data is the

a) range
b) mode
c) standard deviation
d) variance
e) Interquartile Range
f) median

A

e) Interquartile Range

https://statisticsbyjim.com/basics/skewed-distribution/

https://statisticsbyjim.com/basics/variability-range-interquartile-variance-standard-deviation/

A measure of variability is a summary statistic that represents the amount of dispersion in a dataset. How spread out are the values? While a measure of central tendency describes the typical value, measures of variability define how far away the data points tend to fall from the center.

In statistics, variability, dispersion, and spread are synonyms that denote the width of the distribution. Just as there are multiple measures of central tendency, there are several measures of variability.

When a distribution has lower variability, the values in a dataset are more consistent. However, when the variability is higher, the data points are more dissimilar and extreme values become more likely. Consequently, understanding variability helps you grasp the likelihood of unusual events.

> Range is easy to understand, it is based on only the two most extreme values in the dataset, which makes it very susceptible to outliers. If one of those numbers is unusually high or low, it affects the entire range even if it is atypical.

> The interquartile range is the middle half of the data. To visualize it, think about the median value that splits the dataset in half. The interquartile range is the middle half of the data that is in between the upper and lower quartiles. In other words, the interquartile range includes the 50% of data points that fall between Q1 and Q3

> The interquartile range is a robust measure of variability in a similar manner that the median is a robust measure of central tendency. Neither measure is influenced dramatically by outliers because they don’t depend on every value. Additionally, the interquartile range is excellent for skewed distributions, just like the median.

> when you have a normal distribution, the standard deviation tells you the percentage of observations that fall specific distances from the mean. However, this doesn’t work for skewed distributions, and the IQR is a great alternative.

> Variance is the average squared difference of the values from the mean. Unlike the previous measures of variability, the variance includes all values in the calculation by comparing each value to the mean. To calculate this statistic, you calculate a set of squared differences between the data points and the mean, sum them, and then divide by the number of observations. Hence, it’s the average squared difference.

> While higher values of the variance indicate greater variability, there is no intuitive interpretation for specific values. Despite this limitation, various statistical tests use the variance in their calculations. For an example, read my post about the F-test and ANOVA. While it is difficult to interpret the variance itself, the standard deviation resolves this problem!

> The standard deviation is the standard or typical difference between each data point and the mean. When the values in a dataset are grouped closer together, you have a smaller standard deviation. On the other hand, when the values are spread out more, the standard deviation is larger because the standard distance is greater

> The standard deviation is just the square root of the variance. Recall that the variance is in squared units. Hence, the square root returns the value to the natural units. The symbol for the standard deviation as a population parameter is σ while s represents it as a sample estimate. To calculate the standard deviation, calculate the variance as shown above, and then take the square root of it. Voila! You have the standard deviation!

> People often confuse the standard deviation with the standard error of the mean. Both measures assess variability, but they have extremely different purposes.

> When you have normally distributed data, or approximately so, the standard deviation becomes particularly valuable. You can use it to determine the proportion of the values that fall within a specified number of standard deviations from the mean. For example, in a normal distribution, 68% of the values will fall within +/- 1 standard deviation from the mean. This property is part of the Empirical Rule. This rule describes the percentage of the data that fall within specific numbers of standard deviations from the mean for bell-shaped curves.

How well did you know this?
1
Not at all
2
3
4
5
Perfectly
95
Q

20.1 A 64-year-old man presenting for elective surgery is on thyroxine 100 mcg daily. His thyroid function tests are: (Thyroid function tests shown). These results are most consistent with

TFTs thryoxine TSH < .05 T4 and T3 completely normal

a) Hypophysectomy
b) Subclinical Hyperthyoirdism
c) Sick euthyroid
d) Toxic Multinodular goitre

A

b) Subclinical Hyperthyoirdism

Subclinical hyperthyroidism: low TSH, normal T3 + T4
Clinical hyperthyroidism: low TSH, high T3, high/normal T4

Subclinical hypothyroidism: high TSH, normal T3 + T4
Clinical hypothyroidism: high TSH, low/normal T3, i T4

Amiodarone: high/normal TSH, low T3 (2o to inhibition of pituitary T4 to T3 conversion)

Sick euthyroid: low TSH, low T3

Hypophysectomy (central hypothyroidism): low/normal TSH/T3/T4

Compliant on thyroxine: normal TSH, high/normal T3, low T4
Non-compliant w thyroxine (pt taking several tabs prior to Dr’s appointment): high TSH, normal T4

How well did you know this?
1
Not at all
2
3
4
5
Perfectly
96
Q

23.1 A 60-year-old woman presents for thrombectomy with left lower leg ischaemia. She has not received any medications since presentation and takes none at home. The sole abnormality on laboratory testing is an activated partial thromboplastin time (APTT) of 52 seconds. The most likely cause of the raised APTT is

a. Cold agglutinins
b. Erroneous reading
c. Lupus anticoagulant
d. Factor VII deficiency
e. Haemophilia A

A

c. Lupus anticoagulant
(normal PT, raised APTT)

Lupus anticoagulant (more likely to be associated with thrombosis than bleeding)

https://www.uptodate.com/contents/image?imageKey=HEME%2F79969

How well did you know this?
1
Not at all
2
3
4
5
Perfectly
97
Q

20.2, 22.2 The modified Aldrete scoring system is used for determining the

a) Predicts difficulty of bag mask ventilation
b) Safety of day surgery
c) Discharge from recovery
d) Modification of recovery criteria
e) Discharge from hospital

A

c) Discharge from recovery

Aldrete score, which includes five elements (activity, respiration, circulation, consciousness, oxygen saturation) [16].

The original scoring system was developed before the invention of pulse oximetry and used the patient’s colouration as a surrogate marker of their oxygenation status. A modified Aldrete scoring system was described in 1995 which replaces the assessment of skin colouration with the use of pulse oximetry to measure SpO2.

The Modified Aldrete system includes five additional elements that are particularly useful during the Phase II recovery period prior to discharge to home (dressing, pain, ambulation, feeding, urine output)

98
Q

20.2 The most common cause of post operative visual loss after spinal surgery is

a) Corneal abrasion
b) Retinal artery occlusion
c) Central retinal vein occlusion
d) Ischaemic optic neuropathy
e) Occipital infarct

A

a) Ischaemic optic neuropathy

Postoperative visual loss (POVL) occurs in 1/60 000–1/125 000 operations. Spinal surgery has the highest incidence of POVL.

American Society of Anesthesiologists (ASA) Post Operative Visual Loss Registry, spinal surgery accounted for 93/131 (70%) of all cases of visual loss after non-ophthalmic surgery.
Of these:
> 83 were attributable to ischaemic optic atrophy (ION)
> 10 were caused by central retinal artery occlusion (CRAO).

CRAO
- caused by direct pressure on the globe causing raised intraocular pressure and compromising retinal perfusion.
- visual loss is usually unilateral and associated with other signs of pressure (e.g. ophthalmoplegia, ptosis, or altered sensation in the territory of the supraorbital nerve).
- Initial careful positioning of the head and regular checks throughout the procedure in case of movement minimizes the risk
- documentation of eye checks should occur every 30mins and horseshoe shaped head rests should be avoided in prone patients

ION
> associated with:
- male gender
- obesity
- increasing blood loss
- operative procedures >6 hrs in length.
- The use of the Wilson frame has also been implicated.
> final common pathway is thought to be hypoperfusion of the optic nerve, there is no clear association with either intraoperative systemic hypotension or with the presence of peripheral vascular disease or diabetes.
> recently updated ASA practice advisory for POVL associated with spinal surgery recommends regular intraoperative testing of haemoglobin concentration. However, it was unable to suggest a transfusion threshold that would prevent POVL.

Other possible causes of POVL:
1. Cortical ischaemia
2. Haemorrhage into a cerebral tumour.

In high-risk cases, assessment of vision should be performed as soon as possible in PACU and an early ophthalmic opinion sought if there is a suggestion of visual compromise.

Initial management
1. optimization of arterial pressure
2. oxygenation
3. correction of anaemia.

Treatment with agents such as acetazolamide has not been beneficial and there is rarely any useful improvement in vision with either injury, so attention should be focused on preventative measures:
1. Careful positioning with the head at the same level as the heart
2. Meticulous haemostasis,
3. Possibly staging prolonged procedures should be considered.

Because of the devastating nature of this complication, patients should be informed of an increased incidence of visual loss after spinal operations that are expected to be of prolonged duration and associated with significant blood loss.

99
Q

21.2 Of the following drugs, the least likely to cause pulmonary vasodilation when used at low
doses in patients with chronic pulmonary hypertension is

a) Dopamine
b) Dobutamine
c) Vasopressin
d) Milrinone

A

dopamine

  • least likely to cause pulmonary vasodilation (all the others do to my knowledge)
  • From UP TO DATE:
    > At low doses of 1 to 3 mcg/kg per min, dopamine acts primarily on dopamine-1 receptors to dilate the renal and mesenteric artery beds
    > At 3 to 10 mcg/kg per min (and perhaps also at lower doses), dopamine also stimulates beta-1 adrenergic receptors and increases cardiac output, predominantly by increasing stroke volume with variable effects on heart rate.
    > At medium-to-high doses, dopamine also stimulates alpha-adrenergic receptors, although a small study suggested that renal arterial vasodilation and improvement in cardiac output may persist as the dopamine dose is titrated up to 10 mcg/kg per min
    *clinically, the haemodynamic effects of dopamine demonstrate individual variability

Dobutamine (inodilator):
- selective β1-agonist that increases cardiac contractility and reduces pulmonary vascular and systemic vascular resistances

Vasopressin:
- vasopressin may have pulmonary vasodilatory effects in addition to a systemic vasoconstrictive effect

Milrinone (inodilator):
- the phosphodiesterase-3 inhibitors, milrinone and enxoimone, have positive inotropic effects combined with the capacity to reduce RV afterload (‘inodilators’) without significant chronotropic effect, but they can be associated with significant systemic hypotension

100
Q

23.1 In patients with primary adrenal insufficiency, a markedly elevated renin is most likely due to

A Insufficient corticosteroid replacement
B Insufficient fludrocortisone replacement
C Excessive corticosteroid replacement
D Excessive fludrocortisone replacement

A

b. Insufficient fludrocortisone replacement

In Primary Adrenal Insufficency, cortisol deficiency results in decreased feedback to the HPA axis, leading to increased secretion of ACTH to stimulate the adrenal cortex. Simultaneously, MCs deficiency causes increased release of renin by the juxtaglomerular apparatus of the kidneys.

101
Q

22.1 A 30-year-old parturient presents in labour. She has a history of Addison’s disease from autoimmune adrenalitis and has been taking prednisolone 6 mg daily for ten years. On presentation the patient is given hydrocortisone 100 mg intravenously. The most appropriate steroid replacement regimen the patient should receive during labour is

a. 25mg TDS hydrocortisone
b. 8mg/hr hydrocortisone
c. 6mg PO prednisone

A

8mg/hr

Guidelines for mx of glucocorticoids during the perioperative period for patients with adrenal insufficiency

https://associationofanaesthetists-publications.onlinelibrary.wiley.com/doi/10.1111/anae.14963

102
Q

21.2 The oral morphine equivalent of tapentadol 50 mg (immediate release) is

a) 5mg
b) 10mg
c) 15mg
d) 20mg
e) 25mg

A

c) 15mg

Oral Tapentadol 25mg = 8mg Oral Morphine

Oral Oxycodone 5mg = 8mg Oral Morphine

Oral Tramadol 25mg = Oral Morphine 5mg

Oral Hydromorphone 4mg = Oral Morphine 20mg

S/L Buprenorphine 200mcg = 8mg Oral Morphine

IV Oxycodone 5mg = Oral Morphine 15mg

IV Morphine 5mg = Oral Morphine 15mg

IV Hydromorphone 1mg = Oral Morphine 15mg

103
Q

20.1 The substance that should be avoided in a patient with history of anaphylaxis to MMR vaccine is

a) Protamine
b) Penicillin
c) Sulphonamides
d) Gelofusine

A

gelofusin

Anaphylaxis after vaccination is probably due to anaphylactic sensitivity to gelatin or neomycin, not an egg allergy

104
Q

21.1 The implemention of comprehensive multidisciplinary geriatric assessments in the peri-operative period has been shown to

a) Reduce mortality
b) Reduce AKI
c) Reduce periop risk of MACE
d) Reduce length of stay
e) Increase cancellation for surgery

A

d) Reduce length of stay

less time in aged care and reduced mortality

Blue book 2019:
“A referral to a geriatrician for further assessment and management may also be warranted in the preoperative period.
Indeed, a meta-analysis of perioperative interventions to reduce delirium found that a geriatrics consultation before surgery was one of only two perioperative interventions that were associated with a reduction in delirium.”

Association of anaesthetists: The impact of pre-operative comprehensive geriatric assessment on postoperative outcomes in older patients undergoing scheduled surgery: a systematic review

  • reduced medical complications
    > reduced postop delirium,
    > reduced pneumonia
    > reduced pressure sores
  • fewer cancellations
  • reduced length of stay
105
Q

The amount of intravenous potassium chloride required to raise the plasma potassium level from 2.8 mmol/L to 3.8 mmol/L in a normal adult is approximately

a. 10mmol
b. 20mmol
c. 30mmol
d. 100mmol
e. 200mmol

A

e. 200mmol

K+ < 3.0 mmol/L: 200-400 mmol of potassium are required to raise it by 1 mmol/L
K+ > 3.0 mmol/L: 100-200 mmol of potassium are required to raise it by 1 mmol/L

Hypokalaemia P. GLOVER
https://www.cicm.org.au/CICM_Media/CICMSite/CICM-Website/Resources/Publications/CCR Journal/Previous Editions/September 1999/05-Sept_1999_Hypokalaemia.pdf

If the serum potassium level is greater than 3 mmol/L, 100-200 mmol of potassium are required to raise it by 1 mmol/L; 200 - 400 mmol are required to raise the serum potassium level by 1 mmol/L when the potassium concentration is less than 3mmol/L, assuming a normal distribution between cells and the intracellular space, and a linear relationship between plasma potassium and body deficit (which has been described, i.e. 0.27 mmol/L/100 mmol deficit/70 kg), exists. The rate of administration of potassium will be influenced by the presence and seriousness of the pathophysiological changes caused by hypokalaemia. The underlying disorder should also be treated simultaneously.

106
Q

21.1 A five-year-old child weighing 25 kg is to be strictly nil by mouth overnight following a laparotomy. The most appropriate fluid prescription is

a. 65ml/hr N Saline
b. 45ml/hr N saline
c. 45ml/hr N Saline w 5% dex
d. 65ml/hr .45% saline w 2.5% dex
e. 65ml/hr .45% saline w 5% dex

A

b. 45ml/hr N saline w 5% dextrose
Nsaline + 5% dextrose is fluid of choice

A guide to paediatric anaesthesia fluid management
-421 rule overestimates fluid resus
-due to stress response from ADH release
-post-op fluid maintenance is 2/3rds calculated due to increased ADH
-never use hypotonic fluids

https://www.rch.org.au/clinicalguide/guideline_index/Intravenous_fluids/

107
Q

22.2 A 48-year-old man is day two post-laparoscopic high anterior resection. He has used 42 mg of intravenous morphine in the past 24 hours. You wish to start him on oral tapentadol immediate release. The most appropriate equianalgesic dosage would be

a) 50mg six times a day
b) 100mg six times a day
c) 200mg six times a day
d) 300 mg six times a day

A

a) 50mg six times a day

42mg IV Morphine = 126mg Oral Morphine

126/8= 15.75
15.75 x 25 = 393.75 (*400mg/day Tapentadol)

Option 50mg 6 times a day = 300mg
As direct OME to tapentadol conversion is 400mg, a 300mg dose represents a 25% dose reduction, which is line with a 25-50% dose reduction due to incomplete cross-tolerance during opioid rotation.

Oral Tapentadol 25mg = 8mg Oral Morphine

Oral Oxycodone 5mg = 8mg Oral Morphine

Oral Tramadol 25mg = Oral Morphine 5mg

Oral Hydromorphone 4mg = Oral Morphine 20mg

S/L Buprenorphine 200mcg = 8mg Oral Morphine

IV Oxycodone 5mg = Oral Morphine 15mg

IV Morphine 5mg = Oral Morphine 15mg

IV Hydromorphone 1mg = Oral Morphine 15mg

108
Q

21.1 A 45-year-old man has the following results on his blood biochemistry testing (Liver function tests shown). The most likely diagnosis is

a. Cholecystitis
b. Metastatic liver disease
c. Hepatitis C
d. Chronic liver disease
e. Paracetamol toxicity

A

a. Cholecystitis

Example and explanation taken from RACGP:
The raised AlP relative to Alt suggests cholestasis and the high GGt confirms liver origin. The mild hyperbilirubinaemia confirms the clinical impression of jaundice. Biliary disease is highly likely with gallstones the most likely differential diagnosis. however, this clinical picture may also occur in drug reactions or infiltrative conditions. After a careful history, abdominal ultrasound is the most appropriate next investigation.

109
Q

20.2 A 55 year old man with no past history of ischaemic heart disease is 3 days post total hip replacement surgery. He has an episode of chest pain that sounds ischaemic, began at rest and lasts thirty minutes before resolving fully. There are no ECG changes nor troponin rise. The diagnosis is

a. No diagnosis made
b. Unstable angina
c. STEMI
d. NSTEMI
e. MINS

A

b. Unstable angina

Not a Repeat, no Tropnin rise in this question making the answer unstable angina as opposed to NSTEMI

UTD:

Unstable angina (UA) and acute non-ST elevation myocardial infarction (NSTEMI) differ primarily in whether the ischemia is severe enough to cause sufficient myocardial damage to release detectable quantities of a marker of myocardial injury (troponins):

●UA is considered to be present in patients with ischemic symptoms suggestive of an ACS and no elevation in troponins, with or without electrocardiogram changes indicative of ischemia (eg, ST segment depression or transient elevation or new T wave inversion).

●NSTEMI is considered to be present in patients having the same manifestations as those in UA, but in whom an elevation in troponins is present.

MINS: Myocardial injury after non-cardiac surgery (up to 30 days post-op):
1. Elevated postop troponin
2. Resulting from myocardial ischaemia (i.e. no evidence of a non-ischaemic aetiology), not requiring an ischaemic feature (i.e. no chest pain, no ECG change)

VISION studies (Vascular Events in Noncardiac Surgery Patients Cohort Evaluation) demonstrated that severity of MINS strongly associated with 30-day mortality after NCS.

hs-cTnT
<20ng/L ~ 0.5% 30 day mortality
20-64ng/L ~3% 30 day mortality
65-999 ng/L ~9% 30 day mortality
>1000ng/L ~30% 30 day mortality

Whilst VISION trial identified MINS in at risk patients, the question now becomes what interventions are available to prevent this complication?

110
Q

23.1 A five-year-old child weighing 25 kg is to be strictly nil by mouth overnight following a laparotomy. The most appropriate fluid prescription is

a) 45ml/hr 0.9% NS 2.5% dextrose
b) 65ml/hr 0.9% NS 5% dextrose
c) 45ml/hr 0.45% saline with 2.5% dextrose
d) 65ml/hr 0.45% saline with 5% dextrose
e) 45ml/hr 0.9% NS 5% dextrose

A

e. 45ml/hr 0.9% NS 5% dextrose

REPEAT
2/3rd standard full maintenance as unwell

111
Q

21.1 The 12 lead ECG shown (ECG with ST depression V1-V5, perhaps 1mm ste in lead 3) is most consistent with acute total occlusion of the

A. Posterior descending
B. RCA
C. LAD
D. OM

Note this is not the exact ECG with lead I changes too
A

C. LAD

112
Q

21.1 The muscle or muscle group with the greatest sensitivity to the action of non-depolarising neuromuscular blocking agents is/are the

a. Abdominal muscles
b. Adductor pollicis
c. Pharyngeal muscles
d. Diaphragm

A

c. Pharyngeal muscles

Millers Anaesthesia:
Reference artyicle from Millers: https://pubs.asahq.org/anesthesiology/article/92/4/977/710/The-Incidence-and-Mechanisms-of-Pharyngeal-and

An adductor pollicis TOF ratio of 0.90 or less was associated with impaired pharyngeal function and airway protection, resulting in a four- to fivefold increase in the incidence of pharyngeal dysfunction causing misdirected swallowing. Moreover, pharyngeal function and airway protection may be impaired, even if the adductor pollicis muscle has recovered to a TOF ratio of more than 0.90.

113
Q

22.1 Complications of severe anorexia nervosa (body weight < 40% ideal) include all of the following EXCEPT

a. HypoK
b. Cl abnormality
c. Delayed gastric emptying
d. Hypercalcaemia
e. Cardiomyopathy

A

Hypercalcaemia

114
Q

22.1 A 45-year-old man presents on the day of surgery for an elective inguinal hernia repair. He is well
but is noted to be mildly jaundiced. He takes simvastatin for hyperlipidaemia and has no other medical history. He consumes about three standard drinks of alcohol per day and does not smoke. He briefly experimented with illicit drugs more than ten years ago. His laboratory results show: (supplied) The most likely diagnosis is

Normal electrolytes
ALP 85 N
ALT 31 N
AST 31 N
GGT 15 N
Urea 10 [4-9]
Creatinine 103 N
Total protein 74 N
Albumin 40 N
BSL 4.2 N
Bilirubin 29 [0-20]
Conjugated 5
Unconjugated 24

A. Fatty liver
B. Hepatitis
C. Cholestasis
D. Gilbert syndrome
E. Drug induced

A

Gilberts

Gilbert’s syndrome is a benign genetic condition that commonly presents as incidental
hyperbilirubinaemia or painless jaundice.

It is relatively common with a population frequency of approximately 2–10%.

Gilbert’s syndrome is caused by defective bilirubin clearance by the hepatic conjugating enzyme UDP-glucuronosyltransferase

https://www.rcpa.edu.au/getattachment/8b9a8acf-f7f5-4088-951c-3f65f0c2f8fe/Interpreting-liver-function-tests.aspx

115
Q

22.2 Predictors of difficult sedation (agitation or inability to complete the procedure) of patients undergoing gastroscopy do NOT include

A

Unknown options but…

Factors associated WITH difficulty during Gastroscopy were younger age, procedure indication, male sex, presence of a trainee, psychiatric history and benzodiazepine and opioid use. Factors associated with difficulty during COLONOSCOPY were younger age, female sex, BMI <25, procedure indication, tobacco, benzodiazepine, opioid and other psychoactive medication use

116
Q

23.1 A patient with severe abdominal trauma develops acute respiratory distress syndrome. A diagnosis of abdominal compartment syndrome is confirmed if the patient also has a sustained intraabdominal pressure greater than

A. 10mmHg
B. 16mmHg
C. 20mmHg
D. 24mmHg

A

c) 20mmHg

Intra-abdominal hypertension is defined as a sustained intra-abdominal pressure of >12 mm Hg, and abdominal compartment syndrome occurs at a pressure >20 mm Hg in association with new organ dysfunction.

Intra-abdominal hypertension is graded as follows: Grade 1=12–15 mm Hg; Grade 2=16–20 mm Hg; Grade 3=21–25 mm Hg; and Grade 4 >25 mm Hg.

https://academic.oup.com/bjaed/article/12/3/110/258792

117
Q

23.1 Features of hypocalcaemia include all of the following EXCEPT

a. Polydipsia
b. Circumoral tingling
c. Long QTc
d. Laryngospasm
e. Hallucinations

A

a) polydipsia

Hypocalcemia varies from a mild asymptomatic biochemical abnormality to a life-threatening disorder. Acute hypocalcemia can lead to paresthesia, tetany, and seizures (characteristic physical signs may be observed, including Chvostek sign, which is poorly sensitive and specific of hypocalcemia, and Trousseau sign).

https://bestpractice.bmj.com/topics/en-us/160

118
Q

22.1 A man has symptomatic carbon monoxide poisoning. His pulse oximetry (SpO2) and arterial blood gas (PaO2) would be expected to show

a. Normal SpO2, Normal PaO2
b. Normal SpO2, reduced PaO2
c. Reduced SpO2, normal PaO2
d. Reduced SpO2, reduced PaO2

A

a. Normal SpO2, Normal PaO2

ABG

HbCO (elevated levels are significant, but low levels do not rule out exposure)
lactate (tissue hypoxia)
PaO2 should be normal, SpO2 only accurate if measured (not calculated from PaO2)
MetHb (exclude)

https://litfl.com/carbon-monoxide-poisoning/

119
Q

During a routine preoperative examination of a patient’s heart, you note exaggerated splitting of the second heart sound with inspiration. This is characteristically heard in

A. Aortic Reguritation
B. HOCM
C. Left bundle branch block
D. Mitral Stenosis
E. Pulmonary Stenosis

A

E. Pulmonary Stenosis

DERANGED PHYSIOLOGY:

Splitting of the first heart sound
Right bundle branch block can produce a split first heart sound - because the contraction of the right ventricle is delayed- the conduction occurs via the left ventricle rather than the bundle of His- and thefore the closure of the tricuspid valve occurs after a substantial delay.
Atrial septal defect can result in a fixed split of the first heart sound

Splitting of the second heart sound

It is normal for this sound to be split. The high pressure in the systemic circulation slams the aortic valve shut rather abruptly, almost angrily. In contrast, low pressure of the pulmonary circulation tends to close the pulmonary valve gently, and therefore the pulmonary component of the second heart sound (P2) is usually delayed by about 20-30 milliseconds.

It is also normal for increased right ventricular filling to cause a widening of the split. The more blood in the RV, the longer it takes to eject, and therefore the greater the delay until pulmonary valve closure.

n the spontaneously breathing patient, the delay is greatest during inspiration. Naturally, in the patient ventilated with positive pressure the delay is greatest during expiration (positive pressure being a barrier to diastolic filling).

Increased normal splitting of S2

Anything that delays the end of right ventricular systole can cause this sort of picture.

Right bundle branch block - the delay in conduction via the left ventricle causes a delay in right ventricular contraction, and therefore a delay in pulmonary valve closure. The S1 will also be split.
Ventricular septal defect - because the right ventricle receives a large volume load directly from the left ventricle, and therefore takes longer to complete its systolic contraction.

Pulmonary valve stenosis - because the right ventricle takes longer to empty though a narrowed valve

Mitral regurgitation- not because right ventricular contraction is delayed, but because left ventricular contraction is shortened (as the LV empties in both the aortic and the atrial directuion, systole is over very quickly).

Fixed splitting of S2

Atrial septal defect - the atria, joined by a gaping hole in their seput, act as one atrium. The result is a reasonably equal distribution in volume betweent the right and left atrium. This way, both sides of the circulation share the same diastolic filling pressure. Dragging more volume into the right atrium with respiratory activity will not cause an inequality of ventricular filling (between the right and left ventricles) because the venous return will be “shared”.

Reversed splitting of S2

In this situation, P2 occurs before A2, and splitting widens during expiration (or inspiration in the mechanically ventilated patient). This only happens if the conduction to the left ventricle is delayed, or if the left ventricle is massively volume overload (and the right ventricle is not).
Left bundle branch block - the left ventricle depolarises after the right ventricle, and A2 is delayed
Aortic stenosis - the left ventricle empties slowly though a narrow valve
Large patent ductus arteriosus - the left ventricle receives a backflow of blood from the aorta, which causes it to become volume-overloaded

120
Q

23.1 According to the categorisation system used in Australia and New Zealand for prescribing medicines safely in pregnancy, category X denotes drugs which are

a. Drugs that absolutely must not be used for pregnancy. (absolute contraindication)
b. Untested drugs in pregnancy
c. Drugs safe in pregnancy

A

a. Drugs that absolutely must not be used for pregnancy. (absolute contraindication)

https://www.tga.gov.au/australian-categorisation-system-prescribing-medicines-pregnancy

121
Q

22.1 Following the initial subarachnoid haemorrhage from a ruptured aneurysm, the patient is at greatest risk of rebleeding during the following

a. 1-3 days
b. 3-5 days
c. 5-7 days
d. 7-10 days

A

a. 1-3 days

122
Q

22.2 A child with well controlled dysrhythmias has an ASA (American Society of Anesthesiologists) Physical Status classification of at least

a) I
b) II
c) III
d) IV
e) V

A

B II

ASA II Paediatric examples: Asymptomatic congenital cardiac disease, well controlled dysrhythmias, asthma without exacerbation, well controlled epilepsy, non-insulin dependent diabetes mellitus, abnormal BMI percentile for age, mild/moderate OSA, oncologic state in remission, autism with mild limitations

123
Q

23.1 A third heart sound at the apex may be heard in

a) pulmonary stenosis
b) pulmonary hypertension
c) pericarditis
d) pregnancy

A

d. pregnancy

124
Q

23.1 The success rate of stopping smoking before surgery is NOT improved by

a) Bupropion
b) Clonidine
c) Nortroptyline
d) Varencicline
e) SSRI

A

E - SSRIs

ANZCA PG12 Background Paper

125
Q

21.2 A patient presents for a trans-urethral resection of the prostate (TURP). He had a single drug-eluting coronary stent for angina pectoris inserted six months ago and is taking clopidogrel and aspirin. The most appropriate preoperative management of his medications is to

a) Cease aspirin, continue clopidogrel
b) Cease aspirin for 10 days, cease clopidogrel for 5 days
c) Cease clopidogrel for 5 days, continue aspirin
d) Cease clopidogrel for 10 days, continue aspirin
e) Continue both aspirin and clopidogrel

A

c) Cease clopidogrel for 5 days, continue aspirin
- prostatic surgery, the risk of major bleeding may be greater than the risk of stent thrombosis
- For clopidogrel, we stop five days before surgery
- Clopidogrel, if stopped, should be restarted with a loading dose of 300 mg as soon as possible after surgery, perhaps later in the day if postoperative bleeding has stopped. Some experts recommend a higher loading dose of 600 mg to decrease time to effectiveness in the higher-risk postoperative setting
- suggest that surgery be performed in centers with 24-hour interventional cardiology coverage

UP TO DATE: Noncardiac surgery after PCI

Nonemergency noncardiac surgery — For patients who have undergone previous stenting with either BMS or DES and who will need cessation of one or both antiplatelet agents, we prefer to defer planned nonemergency, nonurgent noncardiac surgery until at least six months after stent implantation. The risks of noncardiac surgery before six months are increased after both BMS and DES.
For patients whose surgery requires cessation of one or both antiplatelet agents and cannot wait six months, and where the risks of delaying surgery outweigh the benefits, our recommended minimal duration of DAPT is four to six weeks, depending on the urgency of surgery and risk of thrombotic complication. This is based in part on evidence suggesting that the increased risk of MI and cardiac death is highest within the first month after stent placement and no clear difference in risk between BMS and DES. Although we prefer to wait at least six weeks when possible, in patients for whom earlier surgery is in their best interest after weighing risks and benefits, we sometimes refer patients as early as four weeks after stent placement.

The proinflammatory and prothrombotic risks of surgery may increase the baseline risk of stent thrombosis even in the presence of DAPT and regardless of stent type during this early period after stenting. We believe this risk to be higher prior to the minimum duration of DAPT recommended above, but the final decision to continue or discontinue antiplatelet therapy in the perioperative period should be made only after an informed discussion among the surgeon, managing cardiologist (and other health care providers), and patient has taken place. In many cases, DAPT can be continued in the perioperative period, although for some surgeries, such as neurosurgery, posterior eye surgery, or prostatic surgery, the risk of major bleeding may be greater than the risk of stent thrombosis.

In these patients who undergo noncardiac surgery before the recommended minimum duration of DAPT, a platelet P2Y12 receptor blocker should be discontinued for as brief a period as possible. Aspirin should be continued through the perioperative period, since the risk of stent thrombosis is further increased with the cessation of both aspirin and clopidogrel and surgery can usually be safely performed on aspirin. The rationale to continue aspirin comes in part from the POISE-2 trial (PCI subgroup analysis), which is discussed separately. However, as many neurosurgical patients, for whom bleeding might be life threatening or lead to severe adverse outcomes, were not enrolled in POISE-2, the optimal strategy is not known.

●Minor surgical and dental procedures usually do not require cessation of antiplatelet therapy.
●With regard to stopping P2Y12 inhibitor prior to noncardiac surgery, we generally follow recommendations found in the manufacturer’s package insert for each drug.
- For clopidogrel, we stop five days before surgery; that is, the last dose is taken on the sixth day before surgery.
- For prasugrel, we stop seven days before surgery.
- For ticagrelor, we stop three to five days before surgery.
- Some experts are willing to recommend shorter discontinuation periods for procedures less likely to be associated with major bleeding.
●Clopidogrel, if stopped, should be restarted with a loading dose of 300 mg as soon as possible after surgery, perhaps later in the day if postoperative bleeding has stopped. Some experts recommend a higher loading dose of 600 mg to decrease time to effectiveness in the higher-risk postoperative setting.
●We suggest that surgery be performed in centers with 24-hour interventional cardiology coverage

126
Q

23.1 A patient presents for a transurethral resection of the prostate (TURP). He had a single drug-eluting coronary stent for angina pectoris inserted six months ago and is taking clopidogrel and aspirin. The most appropriate preoperative management of his medications is to

a) Cease aspirin, continue clopidogrel
b) Cease aspirin for 10 days, cease clopidogrel for 5 days
c) Cease clopidogrel for 5 days, continue aspirin
d) Cease clopidogrel for 10 days, continue aspirin
e) Continue both aspirin and clopidogrel

A

C) Cease clopidogrel for 5 days, continue aspirin

WFSA update document
https://resources.wfsahq.org/wp-content/uploads/uia29-Perioperative-management-of-patients-with-coronary-stents-for-non-cardiac-surgery.pdf

Dual antiplatelet therapy should be continued in all patients with coronary stents presenting for surgery.

However, if there is a high risk of surgical bleeding then clopidogrel should be stopped 5-7 days before surgery and monotherapy with aspirin should be continued.

Clopidogrel should be restarted as soon as possible post surgery. Cessation of aspirin therapy may be considered during intracranial surgery and transuretheral resection of prostrate as these procedures are associated with an increased risk of bleeding, but only after contemplating the risk-benefit ratio.

2014 AHA/ACC guidelines on perioperative medicine don’t give a firm answer except: > 180 days since insertion = proceed (Level II b evidence)

127
Q

22.2 A 50-year-old man has the following pulmonary function test result: (provided). The most consistent diagnosis is
FEV1 68%, FVC 68%, DLCO 91%

a. Pulmonary hypertension
b. pulmonary fibrosis
c. myasthenia gravis
d. sarcoidosis

A

c. myasthenia gravis

128
Q

21.2 A 25-year-old male has continued post operative bleeding after an extraction of an impacted third molar tooth under a general anaesthetic. The patient mentions that his father bruises quite easily. His coagulation screen reveals: (Coagulation tests provided). The most likely diagnosis is

His coagulation screen reveals: Prolonged APTT, Normal PT.

a) Factor V Leiden
b) Haemophilia A
c) Haemophilia B
d) Von willebrand disease

A

d) Von willebrand disease
- autosomal dominant inheritance
- may have normal or prolonged APTT, PT is normal

*Haem A: X-linked recessive disorder; would expect prolonged aPTT, and normal PT
*Haem B: X-linked recessive disorder; would expect normal aPTT and normal PT

Up to date:
Inheritance patterns — Most cases of VWD are transmitted as an autosomal dominant trait; this includes types 1 and 2B, and most types 2A and 2M.

Baseline hemostasis assessment —
Most patients will have a complete blood count (CBC) with platelet count and coagulation studies during the initial evaluation for excessive bleeding or bruising.
●Individuals with VWD generally have a normal CBC and a normal platelet count, with the exception of those with type 2B VWD, most of whom will have mild thrombocytopenia (eg, platelet count 100,000 to 140,000/microL).
●Individuals with VWD may have a normal or prolonged activated partial thromboplastin time (aPTT), depending on the degree of reduction of the factor VIII level. The prothrombin time (PT) is normal in VWD.

Up to date:
●Hemophilia A – Inherited deficiency of factor VIII (factor 8 [F8]); an X-linked recessive disorder.
●Hemophilia B – Inherited deficiency of factor IX (factor 9 [F9]); also called Christmas disease; an X-linked recessive disorder.

Laboratory findings —
Hemophilia is characterized by a prolonged activated partial thromboplastin time (aPTT).
However, the aPTT may be normal in individuals with milder factor deficiencies (eg, factor activity level >15 percent), especially in hemophilia B (factor IX deficiency), where even individuals with moderate disease may have a normal aPTT.
In some individuals with hemophilia A, factor VIII levels may increase with stress, leading to a normalization of the aPTT or mis-categorization of factor levels and disease severity.
In patients with hemophilia, the aPTT corrects in mixing studies, unless an inhibitor is present, which only applies to individuals who have received factor infusions or who have an autoantibody such as a lupus anticoagulant or an acquired factor inhibitor.
Mixing studies that do not show correction of a prolonged aPTT suggest an alternative diagnosis such as an acquired factor inhibitor.
The platelet count and prothrombin time (PT) are normal in hemophilia.
Thrombocytopenia and/or prolonged PT suggest another diagnosis instead of (or in addition to) hemophilia.
Measurement of the factor activity level (factor VIII in hemophilia A; factor IX in hemophilia B) shows a reduced level compared with controls (generally <40 percent).
One exception is an individual with mild hemophilia A who undergoes testing when stressed or pregnant and has a falsely elevated factor level. If this is suspected, factor activity testing should be repeated under conditions of low stress.
The plasma von Willebrand factor antigen (VWF:Ag) is normal in hemophilia.
If VWF:Ag is reduced, this suggests the possibility of von Willebrand disease (VWD) rather than (or in addition to) hemophilia.
Urinalysis is not done routinely, but if performed it may sometimes (but not always) show microscopic or macroscopic hematuria.

129
Q

21.1 Of the following, allergy based on cross reaction to penicillin sensitivity is most likely with

A) Cephazolin
B) ceftriaxone
C) cefapime
D) cefaclor
E) cefoxatin

A

D) Cefaclor

  1. Cephalexin? More so than Cephazolin (no B-lactam)
  2. Cefaclor

Source: UpToDate

130
Q

21.1 The risk of a perioperative respiratory adverse event in a child is least likely to be increased by

A. Asthma
B. infection 3 weeks ago,
C. history of eczema,
D. passive smoking

A

history of eczema

131
Q

21.2 A patient presents with a serum sodium of 110 mmol/L. A feature NOT consistent with a
diagnosis of syndrome of inappropriate antiduretic hormone (SIADH) is

a) Urine osmolality <100mOsm/kg
b) Euvolaemic state
c) Urine Na >40 mmol/L
d) Increased cortisol

A

a) Urine osmolality <100mOsm/kg

DIAGNOSTIC CRITERIA

hypotonic hyponatraemia

urine osmolality > plasma osmolality (<275mOsm/kg) (i.e. concentrated urine despite hypotonic blood)

urinary Na+ > 20mmol/L

normal renal, hepatic, cardiac, pituitary, adrenal and thyroid function

euvolaemia (absence of hypotension, hypovolaemia, and oedema)

correction by water restriction

Source LITFL

132
Q

22.1 A 26-year-old patient presents with exertional syncope. The most likely diagnosis is

a. HOCM
b. Long QT syndrome
c. CCF
d. IHD

A

HOCM: pathopneumonic

A person who has syncope during exertion is more likely to have an obstruction to blood flow (aortic stenosis or hypertrophic cardiomyopathy) or ventricular tachycardia as a cause. On the other hand, syncope after completion of exercise is more likely of reflex origin, such as the common faint.

https://www.uptodate.com/contents/syncope-fainting-beyond-the-basics#:~:text=A%20person%20who%20has%20syncope,such%20as%20the%20common%20faint

133
Q

22.1 Created by the Global Initiative for Chronic Obstructive Lung Disease, the alphabetical GOLD groups A to D are tools for the assessment of chronic obstructive pulmonary disease. These classes are based on

a. Symptoms and exacerbations
b. FEV1
c. FEV1 and exacerbations
d. FEV1/FVC and exacerbations
e. FEV1 and symptoms

A

Sx and exacerbations

FEV1

GOLD ABE assessment tool

134
Q

21.2 A factor that is NOT used to calculate the Child-Pugh score is

a) Albumin
b) Bilirubin
c) INR
d) Creatinine
e) Ascites

A

d) Creatinine

  • Originally devised to predict outcomes in Cirrhotic patients undergoing portosystemic Surgery
  • Assess perioperative risk for patients with liver disease who undergo hepatic or non-hepatic surgery
  • Factors include:
    o Encepahlopathy
    § None +1
    § Mild to moerate + 2
    § Severe +3
    o Ascites
    § None +1
    § Mild to moderate (diuretic responsive) +2
    § Severe (diuretic refractory) +3
    o Bilirubin
    § <2 mg/dl +1
    § 2-3mg/dl +2
    § >3 mg/dl +3
    o Albumin
    § >3.5g/dl +1
    § 2.8-3.5g/dl +2
    § <2.8g/dl +3
    o INR
    § <1.7 +1
    § 1.7-2.3 +2
    § >2.3 +3
  • Class A 5-6 points
    o 1-5yr survival rate 95%
  • Class B 7-9 points
    o 1-5 year survival rate 75%
  • Class C 10-15 points
    1-5 yr survival rate 50%

Original study Mortality rates in patients who undergo abdominal surgeries:
- Class A 10%
- Class B 82%
- Class C 82%

Newer Study mortality rates after surgery:
- Class A 2%
- Class B 12%
- Class C 12%

Drawbacks:
- Subjective measurement of:
o ascites
o encephalopathy
- Does not consider
o Pre-op infection
o Aetiology of cirrhosis
o Surgery type

135
Q

21.1 A patient with a purely metabolic acidosis has a serum bicarbonate of 14 mmol/L and a lactate of 3.8 mmol/L. The expected PaCO2 is

a. 24
b. 29
c. 35
d. 40

A

B. 29

PaCO2= 1.5 x 14 + 8
PaCO2= 21 + 8
PaCO2= 29

Winter’s formula: expected PaCO2 = [1.5 x (serum HCO3)] + [8±2]
if PaCO2 lower, there is a concomitant primary respiratory alkalosis
if PaCO2 higher, there is a concomitant primary respiratory acidosis

136
Q

A 50-year-old man has the following pulmonary function test result. The most consistent diagnosis is

FEV1 98% predicted
FVC 98% predicted
DLCO 48% predicted

a) Asthma
b) Obesity
c) Sarcoidosis
d) Pulmonary hypertension

A

d) Pulmonary hypertension
Normal spirometry + low DLCO

Asthma: obstructive pattern and normal DLCO
Obesity: restrictive pattern and normal DLCO
Sarcoid: restrictive pattern and low DLCO

137
Q

22.1 A patient in atrial fibrillation with a CHA2DS2-VASc score of 2 has presented for elective hip surgery. Warfarin had been ceased for four days preoperatively and on the day before surgery the international normalized ratio (INR) was 2.1. The best course of action at this point is to

a) Postpone surgery
b) Vitamin K 3mg IV
c) Prothrombinex 25IU/kg
d) Cell saver intraop
e) Proceed with surgery

A

Give 3mg of Vitamin K and re-check on day of surgery proceed if INR <1.5 on DOS

138
Q

20.2 The flow volume loop is most consistent with (Flow-volume loop shown)

a) Variable intra-thoracic obstruction
b) Variable extra-thoracic obstruction
c) Lower airway obstruction
d) Fixed upper Airway obstruction
e) Mixed pattern

A

d) Fixed upper Airway obstruction

Fixed upper airway obstruction (can be intrathoracic or extrathoracic): flow limitation and flattening are noted in both the inspiratory and expiratory limbs of the flow-volume loop.

139
Q

23.1 A patient requiring an elective major joint replacement has had a recent stroke. The minimum recommended duration between the stroke and surgery is

a) 3 months
b) 6 months
c) 9 months
d) 12 months

A

c. 9
AHA guidelines

12 Months
But 12 weeks minimum

Although the evidence between surgical timing and stroke risk is limited to only these 2 studies, we suggest that elective noncardiac surgery be deferred at least 6 months after a prior stroke, and possibly as long as 9 months to reduce the risk of perioperative stroke in patients undergoing noncardiac surgery.

Alternatively, patients who stand to gain significant improvements in quality of life with elective surgery may consider waiting only 6 months after a prior stroke

REPEAT

140
Q

23.1 In patients without other comorbidities, bariatric weight loss surgery is indicated when
the body mass index (kg/m2) is greater than

a. 35
b. 40
c. 45
d. 50

A

a. 35

Major updates (2022) to 1991 National Institutes of Health guidelines for bariatric surgery

Metabolic and bariatric surgery (MBS) is recommended for individuals with a body mass index (BMI) 35 kg/m2 , regardless of presence, absence, or severity of co-morbidities.

MBS should be considered for individuals with metabolic disease and BMI of 30-34.9 kg/m2

BMI thresholds should be adjusted in the Asian population such that a BMI 25 kg/m2 suggests clinical obesity, and individuals with BMI 27.5 kg/m2 should be offered MBS.

Long-term results of MBS consistently demonstrate safety and efficacy.

Appropriately selected children and adolescents should be considered for MBS.

https://www.soard.org/article/S1550-7289(22)00641-4/fulltext#:~:text=The%201991%20NIH%20Consensus%20Statement,surgery%20that%20is%20applied%20universally.

141
Q

22.2 Of the following, the congenital condition LEAST commonly associated with obstructive sleep apnoea in children is

A

Hypoplastic mandible (micrognathia) – difficult intubation
§ Pierre Robin sequence
§ Treacher Collins
§ Hemifacial microsomia (Goldenhar syndrome)

Midface hypoplasia – difficult bag-mask ventilation
§ Apert syndrome
§ Crouzon syndrome
§ Pfeiffer syndrome
§ Saethre-Chotzen syndrome

Macroglossia – difficult bag-mask ventilation AND difficult intubation
§ Hurler’s/Hunter’s syndrome (mucopolysaccharidoses)
§ Beckwith-Wiedemann syndrome
§ Down’s syndrome

https://www.frca.co.uk/Documents/250%20The%20Difficult%20Paediatric%20Ai

Mucopolysaccharidoses, Down syndrome, muscular dystrophies, and other neurologic disorders have been associated with obstructive sleep apnea

Prevalence of OSAS.
Genetic Disorder Prevalence of OSAS
Neuromuscular diseases 69.2%
Prader–Willi syndrome 94.7%
Arnold–Chiari syndrome 80%
Achondroplasia 100%
Crouzon syndrome 100%
https://www.ncbi.nlm.nih.gov/pmc/articles/PMC8156845/

https://www.frca.co.uk/Documents/250%20The%20Difficult%20Paediatric%20Ai

142
Q

20.1 A 55-year-old lady scheduled for a transphenoidal hypophysectomy undergoes an oral glucose tolerance test with the following results:

GH normal <10
Time 0, BSL 5.5, GH 30, IGF-1 790 (elevated)
Time 30, BSL 7.6, GH 24
Time 60, BSL 7.2, GH 28
Time 90, BSL 6.5, GH 26
Time 120, BSL 5.8, GH 29

These results are most consistent with a diagnosis of

A. Prolactinoma
B. Acromegaly
C. Cushing’s
D. MEN 2
E. Normal

A

Acromegaly

IGF-2 is consistently elevated

GH should be suppressed by glucose load in healthy
pt.

The continued elevation of GH despite glucose is
suggestive of acromegaly

143
Q

23.1 In children, severe sleep apnoea is suggested by an apnoea-hypopnoea index
greater than

a. 10
b. 15
c. 20
d. 30
e. 40

A

a) 10

144
Q

422.1 The current ANZCA guidelines for preoperative fasting of adult patients state that studies have shown that it is safe to administer

a) unlimited clear fluid 2 hours prior
b) 200ml clear fluid 2 hours prior
c) 300ml clear fluid 2 hours prior
d) 400ml clear fluid 2 hours prior

A

400mls of clear fluids pre op

Safe upper limit - definitely has not not been identified and will vary from patient to patient.

Clear fluids
Water / CHO rich fluids / pulp free fruit juice / clear cordial / black tea and coffee

145
Q

22.1 An asymptomatic 65-year-old male with squamous cell carcinoma of the left lung has been referred for assessment of suitability for lung resection. There is no evidence of spread on computerised tomography scanning. PaCO2, electrocardiogram, full blood count and electrolytes are normal. His SpO2 on room air is 95%. His forced expiratory volume in one second is 2.3 litres (predicted 3.3 litres) and forced vital capacity is 3.4 litres (predicted 4.4 litres). The most appropriate course of action is to

a. Proceed with lobectomy or pneumonectomy
b. Proceed with lobectomy only
c. DLCO testing
d. Lung V/Q scan
e. CPET

A

a. Proceed with lobectomy or pneumonectomy

FEV1 surgical suitability:
- >80% or >2l pneumonectomy
○ no further testing required
- >80% or >1.5l lobectomy
○ no further testing required
- <80% or <2l for pneumonectomy
○ -> calculate ppoFEV1
- <80% or <1.5l for lobectomy
○ -> perform DLCO and express as % of predicted DLCO
○ Saturations on air
- ppoFEV1 < 40% and DLCO <40% = High Risk
- ppoFEV1 >40% and DLCO >40% and SaO2 >90% = Average risk (no further testing)

146
Q

21.2, 20.1 The muscle or muscle group with the greatest resistance to the action of non-depolarising neuromuscular blocking agents is the

a) Adductor pollicis
b) Diaphragm
c) Orbicularis oculi
d) Pharyngeal

A

b) Diaphragm

147
Q

20.1 RFTS: Normal ratio, low FVC, low FEV1, Normal DLCO:
a) Sarcoid
b) Myasthenia Gravis
c) Asthma
d) Emphysema

A

b) Myasthenia Gravis

148
Q

22.2 A 76-year-old man requires an emergency thoracotomy to treat an expanding haemothorax. He is mildly hypotensive and is not fasted. His plasma electrolytes and haemoglobin are below. The most appropriate strategy to employ to intubate him with a double lumen endotracheal tube is to (use)

K 6.3 Ur 7-ish Cr 174

a. Cisatracurium 0.5mg/kg
b. Rocuronium 1.2mg/kg
c. Suxamethonium 1mg/kg
d. Suxamethonium 0.5mg/kg (?was this an option)

A

b. Rocuronium 1.2mg/kg

Cis not appropriate for intubation

Sux with K 6.3 is risky. (I’ve never heard of reduced dose)

149
Q

21.2 In a patient with anaemia of chronic disease, of the following the most likely to be elevated is

a. MCV
b. transferrin saturation
c. Increased soluble Transferrin Receptor
d. Ferritin
e. Total iron binding capacity

A

d. Ferritin

ANZCA blue book:

ACD caused primarily by inflammation

Mechanism:
1. Iron
- Inflammation reduces Iron availabilty as a protective mechanism whereby Iron is sequestered and stored in macrophages to limit availability to microbial pathogens
- Hepcidin expression is increased, this prevents the release of Iron by reticuloendothelial system resulting in “functional iron deficiency” with reduced tissue availability of iron, despite apparently normal total body iron stores. (hence increased Ferritin)

  1. Response to erythropoietin
    - mechanism not clear suspect blunting of response to erythropoietin
  2. Therapeutic agents
    chemotherapies that impair bone marrow response to erythropoiesis
    65% of patients with lung and gynae cancer treated with platinum based drug develop anaemia

RCPA advice on interpretation of Soluble Transferrin Receptor:

Soluble transferrin receptor levels in plasma are elevated if there is increased iron demand due to Iron deficiency, increased erythropoiesis (eg, Haemolysis) or dyserythropoiesis (eg, Megaloblastic anaemia), regardless of other, coexistent states.

Thus, it can be used to demonstrate iron deficiency in patients who also have an acute phase response and it can distinguish Iron deficiency from the Anaemia of chronic disease.

Patients with an acute phase response have reduced plasma iron and transferrin with elevation of Ferritin, making these usual indicators unreliable.

150
Q

20.2 A patient with a purely metabolic acidosis has a serum bicarbonate of 14 mmol/L and a lactate of 3.8 mmol/L. The expected PaCO2 is

A

29mmHg

151
Q

22.2 Suxamethonium may be safely given to patients with (list of neuromuscular diseases given)

a. Becker muscular dystrophy
b. Myaesthenia gravis (new option)
c. Guillain Barre
d. Hypokalaemic periodic paralysis (new option)
e. Duchenne muscular dystrophy

A

b. Myaesthenia gravis

ED95 is 0.8mg/kg in a MG patient

152
Q

20.1 The flow volume loop is most consistent with

a) Variable intra-thoracic obstruction
b) Variable extra-thoracic obstruction
c) Fixed large airway
d) Restrictive lung pattern
e) Mixed pattern

A

b) Variable extra-thoracic obstruction

153
Q

23.1 Expected features of Guillain-Barré syndrome include

A. Descending paralysis
B. Flaccid paralysis
C. Unilateral leg weakness

A

b) flaccid paralysis

Guillain–Barré syndrome (GBS) is an inflammatory disease of the PNS and is the most common cause of acute flaccid paralysis

154
Q

20.2 The ANZCA guidelines regarding pre-operative oral intake for infants under 6 months of age having an elective procedure under anaesthesia are

a) Breast milk 2 hours before, clear fluids 1 hour before to max 3ml/kg
b) Breast milk 2 hours, clear fluids 1 hour before to max 5ml/kg
c) Breast milk 3 hours, clear fluids 1 hour to max 3ml/kg
d) Breast milk 3 hours, clear fluid 1 hour to max 5ml/kg
e) Breast milk 4 hours, clear fluids 1 hour to max 3ml/kg

A

Repeat

c) Breast milk 3 hours, clear fluids 1 hour to max 3ml/kg

Infants <6 months having elective procedure
* 4 hours for formula
* 3 hours for breast milk
* 1 hour for clear fluids (≤3 ml/kg/hr)

Children > 6 months having elective procedure
* 6 hours for limited solid food or formula
* 4 hours for breast milk
* 1 hour for clear fluids (≤ 3ml/kg/hr)

155
Q

21.1 Perioperative overheating is most likely to cause worsening of symptoms of

a) Duchenne Muscular dystrophy
b) Myasthenia gravis
c) Multiple sclerosis
d) Myotonica dystrophia
e) Eaton Lambert syndrome

A

multiple sclerosis.

https://academic.oup.com/bjaed/article/11/4/119/266998
Anaesthetic considerations.
- Local anaesthetics may exacerbate symptoms due to the increased sensitivity of demyelinated axons to local anaesthetic toxicity.
- Non-depolarizing neuromuscular blocking agents may be used in normal doses.
Caution should be exercised when using depolarizing neuromuscular blocking agents if the patient is debilitated.
- Temperature maintenance is important as symptoms can deteriorate with an increase in temperature, as demyelinated axons are also more sensitive to heat.

156
Q

22.2 The 2012 Berlin definition of the acute respiratory distress syndrome (ARDS) defines moderate disease as one with a PaO2 / FiO2 ratio (in mmHg) of

a) 50-100
b) 100-200
c) 200-300
d) 300-400

A

a) 100-200

2012 BERLIN DEFINITION OF ARDS

ARDS is an acute diffuse, inflammatory lung injury, leading to increased pulmonary vascular permeability, increased lung weight, and loss of aerated lung tissue…[with] hypoxemia and bilateral radiographic opacities, associated with increased venous admixture, increased physiological dead space and decreased lung compliance.

Key components
- acute, meaning onset over 1 week or less
- bilateral opacities consistent with pulmonary edema must be present and may be detected on CT or chest radiograph
- PF ratio <300mmHg with a minimum of 5 cmH20 PEEP (or CPAP)
- “must not be fully explained by cardiac failure or fluid overload,” in the physician’s best estimation using available information — an “objective assessment“ (e.g. echocardiogram) should be performed in most cases if there is no clear cause such as trauma or sepsis.

Severity
- ARDS is categorized as being mild, moderate, or severe:

157
Q

20.1 A 22-year-old patient is scheduled for resection of a large extra-adrenal paraganglionoma. The tumour is secreting metanephrine. The most likely therapy to be commenced at the preassessment clinic prior to surgery is

a) Prazocin
b) Phentolamine
c) Magnesium
d) Phenoxybenzamine
e) Ca channel blocker

A

Phenoxybenzamine

UpToDate
Phenoxybenzamine​ is the preferred drug for preoperative preparation to control blood pressure and arrhythmia in most centers in the United States. It is an irreversible, long-acting, nonspecific alpha-adrenergic blocking agent.
With their more favorable side-effect profiles and lower financial cost, selective alpha-1-adrenergic blocking agents (eg, ​prazosin​, t​ erazosin​, or d​ oxazosin​) are utilized in many centers or are preferred to ​phenoxybenzamine​ when long-term pharmacologic treatment is indicated (eg, for metastatic pheochromocytoma).

158
Q

22.2 In Australia and New Zealand, the proportion of blood donors who are cytomegalovirus (CMV) seropositive is
(rough numbers in the options, can’t remember exactly)

a. 65 to 85 per million
b. 650 to 850 per million
c. 6.5 to 8.5 per hundred
d. 65 to 85 per hundred

A

d. 65 to 85 per hundred

85% of australians are CMV positive by the age of 40

https://www.blood.gov.au/system/files/documents/cmv-blood-components.pdf

159
Q

22.1 Abnormal Q waves are NOT a feature of the electrocardiogram in

A. Digitalis toxicity
B. LBBB
C. Recent transmural MI
D. Wolff-Parkinson-White
E. Previous MI

A

A. Digitalis toxicity

Miller’s
The ECG made easy
http://lifeinthefastlane.com/ecg-library/pmi/

Normal Q waves
- Due to depolarisation of the interventricular septum from left to right
- Seen in the left-sided leads (I, aVL, V5, V6)

Pathological Q waves
- > 1 mm depth
- > 1 mm (= 40 ms) across

Digoxin ECG changes
- Therapeutic: prolonged PR interval (AV nodal delay), shortened QTC intervals (rapid ventricular repolarisation), ST depression (↓ slope of phase 3), T wave inversion
- Toxic: atrial or ventricular arrhythmias (↑ automaticity), prolonged PR interval → heart block, SA node inhibition → sinus arrest
- Atrial tachycardia with block = most common arrhythmia attributed to digoxin toxicity
- VF = most frequent cause of death
- QRS = normal!

Q waves in MI
- Occur with transmural infarctions, and are less likely with subendocardial infarctions
- Develops days after the onset of AMI, and is usually permanent
- Indicates the part of the heart that has been damaged

LBBB ECG changes
- Wide QRS
- Wide QS complex in lead V1
- Wide R wave in lead V6 with slight notching at the peak and TWI
- The axis is highly variable: can be normal or deviated to the left or right

Wolff-Parkinson-White syndrome
- Due to the presence of an accessory bundle between the atrium and ventricle, which has no AV node to delay conduction
- Short PR interval
- Early slurred upstroke of the QRS complex due to delta wave

160
Q

23.1 The parameter that changes most with increasing age in the otherwise normal lung is the

a. Closing capacity
b. Residual volume
c. FRC
d. Lung capacity.

A

a) Closing capacity

see graph in Millers

161
Q

21.1 The domains described in the Edmonton Frail Scale do NOT include

a. Cognition
b. Mental illness
c. Weight
d. Age
e. Functional assessment

A

age

Domains:
1. Cognition
2. General health status
3. Functional independance
4. social support
5. Medication use
6. Nutrition
7. Mood ?interpreted as mental illness in stem?
8. Continence
9. Functional performance

Scoring
0-5= Not frail
6-7= Vulnerable
8-9= Mild Frailty
10-11= Moderate Frailty
12-17= Severe Frailty

Easy way to remember is CONFUSION
Cognition
Overall Health
Nutrition
Function performance
Urine incontinence
Social support
Independence
Oral Meds
Nightmares

162
Q

20.2 The flow volume loop is most consistent with (Flow-volume loop shown)

a) Variable intra-thoracic obstruction
b) Variable extra-thoracic obstruction
c) Lower airway obstruction
d) Fixed upper Airway obstruction
e) Mixed pattern

A

a) Variable intra-thoracic obstruction

Dynamic (or variable, nonfixed) intrathoracic obstruction: flow limitation and flattening are noted on the expiratory limb of the loop.

163
Q

23.1 An absolute contraindication to skin prick testing for the diagnosis of allergies is

a. Pregnancy
b. Severe dermatographia
c. Concurrent antihistamine use
d. Concurrent beta blocker
e. Asthma

A

b) severe dermatographia

164
Q

23.1 An absolute contraindication to skin prick testing for the diagnosis of allergies is

a. Pregnancy
b. Severe dermatographia
c. Concurrent antihistamine use
d. Concurrent beta blocker
e. Asthma

A

b) severe dermatographia

165
Q

23.1 An otherwise healthy child with a history of leukaemia four years ago, now in
remission, has an American Society of Anesthesiologists (ASA) classification of at
least

a. 1
b. 2
c. 3
d. 4
e. 5

A

ASA 2

166
Q

21.1 A 50-year-old man is seen prior to his hip revision surgery. His blood results are

Hb 110 (130-170 normal range)
Ferritin 31 (30-100 range)
Transferrin saturation 21% (normal 20-80)
CRP 10 (0.1-10 normal)

The most likely diagnosis is

a) iron deficiency anaemia
b) anaemia of chronic disease
c) anaemia of chronic inflammation
d) anaemia of chronic inflammation with iron deficiency
e) megaloblastic anaemia

A

Anaemia of chronic inflamation with iron deficiciency

167
Q

21.1 Effective pharmacotherapy options to support smoking cessation in the perioperative period include all of the following EXCEPT

a) bupropion
b) clonidine
c) nortoptyline
d) Varenicicline
e) fluoxetine

A

Fluoxetine

168
Q

21.2 You are examining the precordium of a patient in the preadmission clinic and hear a fourth heart sound at the apex. This finding is consistent with

a) AR
b) Athlete
c) Normal
d) Hypertension

A

) Hypertension

Talley & O’Connor CVS Exam:
S3: Physiological in pregnancy; sign of LV failure; AR & MR
S4: Never physiological, most often due to systemic hypertension

Atrial gallop - stiff LV
- hypertrophy or ischaemic ventricle

Source CV phys

169
Q

22.1 The most common type of perioperative stroke is

a. Embolic
b. Hypotensive
c. Thrombotic

A

Emboli

Blue book repeat

170
Q

20.2, 21.2 The anion which contributes the most to the anion gap is

a) Albumin
b) Chloride
c) Phosphate
d) Bicarbonate

A

albumin

ALBUMIN AND PHOSPHATE
the normal anion gap depends on serum phosphate and serum albumin
the normal AG = 0.2 x [albumin] (g/L) + 1.5 x [phosphate] (mmol/L)
albumin is the major unmeasured anion and contributes almost the whole of the value of the anion gap.
every 1g/L decrease in albumin will decrease anion gap by 0.25 mmoles
a normally high anion gap acidosis in a patient with hypoalbuminaemia may appear as a normal anion gap acidosis.
this is particularly relevant in ICU patients where lower albumin levels are common
Effects of albumin:
Anion gap may be underesitmated in hypoalbuminaemia, because if albumin decreased by 1g/L then the anion gap decreases by 0.25 mmol
To overcome the effects of the hypoalbuminaemia on the AG, the corrected AG can be used which is AG + (0.25 X (40-albumin) expressed in g/L

171
Q

20.1 The catheter type most likely to be associated with bloodstream sepsis per days insertion is:

a) Peripheral venous catheter
b) PICC
c) non-tunneled CVL
d) tunneled CVL
e) Peripheral arterial catheter

A

c) non-tunneled CVL

UTD:
In a systematic review that included 200 studies, the risk of catheter-related blood stream infection per 1000 catheter-days and varied by types of intravascular catheter were as follows [38]:

●Peripherally inserted central catheters (PICCs) – 1.1 (95% CI 0.9-1.3)

●Cuffed and tunneled central venous catheters – 1.6 (95% CI 1.5-1.7)

●Noncuffed central venous catheters

*Nonmedicated and tunneled – 1.7 (95% CI 1.2-2.3)

*Nonmedicated and nontunneled – 2.7 (95% CI 2.6-2.9)

However, there was no adjustment for severity of illness. Therefore, a particular type of catheter could be associated with an increased risk of infection if it was preferentially used in more severely ill or vulnerable patients.

172
Q

23.1 The Sequential Organ Failure Assessment (SOFA) score is used in intensive care for the assessment of sepsis. This score does NOT include the

a. Bilirubin
b. Platelets
c. PaO2/FiO2
d. GCS
e. Hypoglycaemia

A

e) hypoglycaemia

173
Q

21.1, 22.2 IIn critically ill patients undergoing mechanical ventilation, energy dense enteral nutrition (1.5 kcal/mL/kg) compared to routine (1 kcal/mL/kg) enteral feeding provides

a) Higher incidence of VAP
b) Lower incidence of AKI
c) Lower all cause 90-day mortality
d) No difference

A

d) No difference

Repeat

Conclusions

In patients undergoing mechanical ventilation, the rate of survival at 90 days associated with the use of an energy-dense formulation for enteral delivery of nutrition was not higher than that with routine enteral nutrition. (Funded by National Health and Medical Research Institute of Australia and the Health Research Council of New Zealand; TARGET ClinicalTrials.gov number, NCT02306746. opens in new tab.)

https://www.nejm.org/doi/full/10.1056/NEJMoa1811687

174
Q

21.2 An adult with renal failure on regular haemodialysis has an ASA (American Society of
Anesthesiologists) physical status classification of at least

a) 1
b) 2
c) 3
d) 4
e) 5

A

ASA 3

Source: ASA Classification
https://www.asahq.org/standards-and-guidelines/asa-physical-status-classification-system

175
Q

21.1 Of the following, the lifestyle modification that is least effective in reducing essential hypertension is

a. Stopping caffeine
b. Low sodium diet
c. Low potassium diet
d. Exercise

A

c. Low potassium diet (high K diet - dec HTN)

176
Q

21.1 Chronic recreational use of nitrous oxide may lead to
a. Anaemia due to decreased EPO
b. Anaemia from glutathione deficiency
c. Neurological damage due to methionine deficit
d. Pulmonary hypertension

A

neurological damage due to methionine deficit

177
Q

21.2 In pulmonary function testing the presence of airflow limitation is defined by a post- bronchodilator FEV1/FVC ratio less than
a) 0.5
b) 0.6
c) 0.7
d) 0.8

A

c) 0.7

178
Q

21.2 Of the following, the LEAST likely cause of high anion gap metabolic acidosis is

a) Pancreatic fistula
b) DKA
c) Cardiac failure
d) Anti-retroviral
e) Methanol

A

pancreatic fistula

-> should cause NAGMA

HAGMA:
Lactate
Toxins
Ketones
Renal failure

NAGMA
Chloride
Addison’s, adrenal insuffiency, acetazolamide
GI loss (pancreatic fistula)
Extra: RTA

Anion gap:
- Anion Gap = Na+ – (Cl- + HCO3-)
- The Anion Gap (AG) is a derived variable primarily used for the evaluation of metabolic acidosis to determine the presence of unmeasured anions
- The normal anion gap is assumed to be 12, and the normal HCO3 is assumed to be 24

Delta ratio:
- can check delta ratio in the presence of a high anion gap metabolic acidosis (HAGMA) to determine if it is a ‘pure’ HAGMA or if there is coexistant normal anion gap metabolic acidosis (NAGMA) or metabolic alkalosis.

179
Q

22.1 A 65-year-old man presents to the preadmission clinic two weeks prior to his total knee replacement. His blood results include haemoglobin 100 g/L, ferritin 20 μg/L and normal C-reactive protein. The best course of action is to

a. Proceed
b. EPO and iron
c. Iron tablet and delay 3 months
d. Iron transfusion and proceed
e. PRBC

A

Postpone 3 months and give oral iron

180
Q

20.2 The flow volume loop is most consistent with (Flow-volume loop shown)

a) Variable intra-thoracic obstruction
b) Variable extra-thoracic obstruction
c) Mixed Pattern
d) Fixed upper Airway obstruction
e) Normal

A

e) Normal

Normal flow-volume loop: the expiratory portion of the flow-volume curve is characterized by a rapid rise to the peak flow rate, followed by a nearly linear fall in flow. The inspiratory curve is a relatively symmetrical, saddle-shaped curve.

181
Q

22.1 The effect of a drop in patient core temperature from 37 C to 34 C is to

a. Increased k time
b. Decreased viscosity
c. Decreased platelet function

A

c. Decreased platelet function

182
Q

21.2 The most common type of perioperative stroke is

a) Thrombotic
b) Ischaemic
c) Hypotension
d) Embolic
e) Haemorrhagic

A

embolic

183
Q

21.2 ANZCA fasting guidelines classify all of the following as clear fluids EXCEPT

a) clear cordial
b) black coffee
c) strained broth
d) pulp free fruit juice

A

strained broth

ANZCA PS07:
“Clear fluids are regarded as water, carbohydrate rich fluids, specifically developed for perioperative use, pulp free fruit juice, clear cordial, black tea and coffee. It excludes fluids containing particulate matter, soluble fibre, milk-based drinks and jelly”

184
Q

21.1 A 50 year old man has the following pulmonary function test result. The most consistent diagnosis is

FEV1 - test result - predicted - % predicted 68%
FVC - test result - predicted - % predicted 68%
DLCO 46%

a) Asthma
b) Myasthenia Gravis
c) Emphysema
d) Sarcoidosis
e) Pulmonary Hypertension

A

d) Sarcoidosis

Pulmonary hypertension: Normal spirometry + low DLCO
Asthma: obstructive pattern and normal DLCO
Obesity: restrictive pattern and normal DLCO
Sarcoid: restrictive pattern and low DLCO

185
Q

21.1, 22.2 Intraoperative lung protective ventilation strategies include all of the following EXCEPT
A. Vt 6-8ml/kg
B. Patient titrated PEEP
C. Recruitment manoeuvre
D. I:E ratio 1:3

A

I:E ration 1:3

BJA Lung-protective ventilation for the surgical patient: international expert panel-based consensus recommendations:

An expert consensus was reached for 22 recommendations and four statements.

The following are the highlights:
(i) a dedicated score should be used for preoperative pulmonary risk evaluation; and
(ii) an individualised mechanical ventilation may improve the mechanics of breathing and respiratory function, and prevent PPCs.

The ventilator should initially be set to a tidal volume of 6–8 ml kg−1 predicted body weight and positive end-expiratory pressure (PEEP) 5 cm H2O.

PEEP should be individualised thereafter.

When recruitment manoeuvres are performed, the lowest effective pressure and shortest effective time or fewest number of breaths should be used.

Inspiratory/expiratory ratio:
Several studies have compared prolonged inspiratory-to-expiratory (I:E) ratios to the 1:2 ratio commonly used during mechanical ventilation.

An I:E ratio of 1:1, which has been characterised as providing a ‘balanced stress to time product’, was associated with attenuation of lung damage.
Prolonged I:E ratio increases mean airway pressure and concomitantly reduces peak airway pressure.

Studies using prolonged inspiratory times have described beneficial effects, including increased CRS and PaO2, lower alveolar–arterial gradient, and reduced inflammatory markers.

Given the lack of evidence for a clear benefit of a specific I:E ratio, no recommendation was offered by the panel.

However, the panel noted that optimisation of inspiratory time for individual patients can be achieved by monitoring parameters, such as oxygenation, CRS, and ΔP.

Intraoperative FIO2

Increased FIO2 during mechanical ventilation is administered to prevent or correct hypoxaemia, but may result in hyperoxia.

The negative effects of hyperoxia are not clear, but it has been suggested that it may increase oxidative stress, peripheral vascular and coronary artery vasoconstriction, decrease cardiac output, increase resorption atelectasis, and increase the rate of PPCs.

Recommendations for optimal use of oxygen and current evidence regarding the association between hyperoxaemia and clinically relevant outcomes during intraoperative mechanical ventilation are lacking.

Few studies have revealed a protective effect of hyperoxaemia, some report an association with mortality, whilst others show no association with clinically relevant outcomes.

Therefore, in the absence of evidence, the most prudent course of action during mechanical ventilation is to maintain normoxaemia.

SpO2 monitoring can assist in the detection of hypoxaemia, but during oxygen therapy SpO2 cannot detect hyperoxia.

Whilst SpO2 monitoring reduces the incidence of hypoxaemia, it does not improve the overall patient outcomes and does not reduce morbidity and mortality.

Therefore, once the airway is secured, FIO2 should be set to ≤0.4 with the goal of using the lowest possible FIO2 to achieve normoxia (or SpO2 ≥94%)

Unnecessarily high FIO2 should be avoided.

Administering lower FIO2 will not only decrease the risk of hyperoxia, but will also reduce the masking effect of oxygen therapy and allow for earlier diagnosis of gas-exchange impairment.

186
Q

22.1 A patient with a body mass index 34 kg/m2 with no other disease has an ASA (American Society of Anesthesiologists) Physical Classification of at least

a. I
b. II
c. III
d. IV

A

b. II

187
Q

21.1 A patient undergoing robotic prostatectomy with controlled mandatory volume ventilation has the following measurements:

plateau pressure 32 cmH2O, extrinsicPEEP 8 cmH2O, autoPEEP 4 cmH2O, peak pressure 38 cmH2O, tidal volume 600mL

The static compliance is

20 ml/cmH20
23 ml/cmH2O
25 ml/cmH20
30 ml/cm H20

A

30ml/cm H2O

600/32-8+4 = 30
Static lung compliance (Cstat), mL/cm H2O = TV / (Plateau pressure (Pplat) – TotalPEEP)

remembered parameters included PEEP = 8 and autop PEEP = 4
if actual answer states TotalPEEP= 8 then no need to add 4 to the calculation

188
Q

21.2 A man with atrial fibrillation has no valvular heart disease. According to joint American Heart Association (AHA), American College of Cardiology (ACC) and Heart Rhythm Society (HRS) guidelines, oral anticoagulants are definitely recommended if his CHA2DS2-VASc score is greater than or equal to

a) 1
b) 2
c) 3
d) 4
e) 5

A

b) 2

  • if male CHA2DS2-VASc score ≥2 to be recommended chronic OAC (Grade 1A).
  • if female CHA2DS2-VASc score ≥3 to be recommended chronic OAC (Grade 1A).
  • non-sex risk factor also holds bearing:
  • For patients with CHA2DS2-VASc score of 1 in males and 2 in females based on age 65 to 74 years, we recommend chronic OAC (Grade 1A).

Up to date:

Our approach to deciding whether to prescribe anticoagulant therapy for patients with AF (excluding those with rheumatic mitral stenosis that is severe or clinically significant [mitral valve area ≤1.5 cm2], a bioprosthetic valve [surgical or bioprosthetic] within the first three to six months after implantation, or a mechanical heart valve) is as follows:

*For a CHA2DS2-VASc score ≥2 in males or ≥3 in females, we recommend chronic OAC (Grade 1A).

*For a CHA2DS2-VASc score of 1 in males and 2 in females:
-For patients with CHA2DS2-VASc score of 1 in males and 2 in females based on age 65 to 74 years, we recommend chronic OAC (Grade 1A). Age 65 to 74 years is a stronger risk factor than the other factors conferring one CHA2DS2-VASc score point.
-For patients with other risk factors, the decision to anticoagulate is based upon the specific nonsex risk factor and the burden of AF. For patients with very low burden of AF (eg, AF that is well documented as limited to an isolated episode that may have been due to a reversible cause such as recent surgery, heavy alcohol ingestion, or sleep deprivation), it may be reasonable to forgo chronic OAC and institute close surveillance for recurrent AF, although it may not be possible to reliably estimate AF burden from surveying symptoms or infrequent monitoring. The frequency and duration of AF episodes vary widely over time, and episodes are often asymptomatic.

*For patients with a CHA2DS2-VASc of 0 in males or 1 in females, we suggest against OAC (Grade 2C). Patient values and preferences may impact the decision. For example, a patient who is particularly stroke averse and is not at increased risk for bleeding may reasonably choose anticoagulation, particularly if the patient is a candidate for treatment with a direct oral anticoagulant (DOAC).

2019 AHA/ACC/HRS Focused Update of the 2014 AHA/ACC/HRS Guideline

189
Q

22.1 Moderate obstructive sleep apnoea in children is diagnosed by an apnoea-hypopnoea index of

a. 5-10
b. 10-15
c. 15-20
d. 20-25
e. 25-30

A

a. 5-10

190
Q

23.1 In a 20-year-old with cystic fibrosis, the most likely finding on pulmonary function
tests is

a. Mixed obstruction and restrictive pattern
b. Restrictive with normal DLCO
c. Restrictive with low DLCO
d. Obstruction with reduced RV
e. Obstructive with reduced FEV1

A

e. Obstructive w/ reduced FEV1

Mucous narrowing airways = obstructive
Parenchymal damage = restrictive

Obstructive PFP remains the most common pulmonary function pattern in adult CF and is associated with
-decrease FEV1 & FVC/FEV1

For patients with CF, an obstructive pattern is generally seen, with a decrease in forced expiratory volume in 1 s (FEV1), and forced vital capacity (FVC) to FEV1 ratio.

https://academic.oup.com/bjaed/article/11/6/204/263786

191
Q

21.1, 22.2 A patient requiring an elective joint replacement has had a recent stroke. The minimum time to wait after the stroke before proceeding with surgery is

a. 3 months
b. 6 months
c. 9 months
d. 12 months

A

c. 9
AHA guidelines

12 Months
But 12 weeks minimum

Although the evidence between surgical timing and stroke risk is limited to only these 2 studies, we suggest that elective noncardiac surgery be deferred at least 6 months after a prior stroke, and possibly as long as 9 months to reduce the risk of perioperative stroke in patients undergoing noncardiac surgery.

Alternatively, patients who stand to gain significant improvements in quality of life with elective surgery may consider waiting only 6 months after a prior stroke

192
Q

21.2 Allergic cross-reactivity between penicillins and cephalosporins is mediated by the

a) R1 side chain
b) R2 side chain
c) Beta lactam ring
d) Imidazole group

A

a) R1 side chain

UP TO DATE:
- sensitisation to R1 side chain in cephalosporins important in determining cross reactivity with penicillins.

193
Q

20.2 A 50 year old man has the following pulmonary function test result.

FEV1 68% predicted,
FVC 68% predicted,
DLCO 46% predicted

The most consistent diagnosis is

a) Asthma
b) Myasthenia Gravis
c) Emphysema
d) Sarcoidosis
e) Pulmonary Hypertension

A

Repeat

d) Sarcoidosis

Normal FEV1/FVC ratio = no obstruction
Low FVC = restrictive pattern
Low DLCO = interstitial lung disease

Asthma and emphysema would have obstructive pattern.
Myasthenia gravis would have normal DLCO
Pulmonary HTN would have normal spirometry and low DLCO.

194
Q

An ASA 1 28-year-old man attends for inguinal hernia repair under general anaesthesia. He is administered propofol 180mg morphine 8mg rocuronium 50mg cephazolin 2g Post induction he develops an erythematous rash on his chest and arms, swelling of his lips and face, and severe hypotension. Preliminary blood results show: (allergy related tests shown).

Tryptase at 1 hour 321 (11)
Tryptase at 3 hours 58 (11)
RAST Morphine 29 (15)
Serum IgE 88 (300)

The most likely diagnosis is

a. Morphine anaphylaxis
b. Rocuronium anaphylaxis
c. Cephazolin Anaphylaxis
d. Propofol Anaphylaxis
e. Opioid related histamine release

A

Answer: b. rocuronium anaphylaxis

NB
RadioAllergoabsorbentSpecificTesting is a serum test for specific IgE antibodies
RAST morphine is both more sensitive and more specific than the RAST for individual NMBDs (due to reaction with quaternary ammonium) and is being used increasingly to determine NMBDs as cause of anaphylaxis. IKR!

http://www.anzaag.com/anaphylaxis-management/testing-guidelines.pdf

195
Q

A patient with known suxamethonium allergy is most likely to demonstrate cross reactivity with

a. Mivacurium
b. Cisatracurium
c. Atracurium
d. Rocuronium
e. Cephazolin

A

Answer: d. Rocuronium

BJA Anaphylaxis to neuromuscular blocking drugs: incidence and cross-reactivity in Western Australia from 2002 to 2011
https://academic.oup.com/bja/article/110/6/981/245571

Rocuronium has a higher rate of IgE-mediated anaphylaxis compared with vecuronium, a result that is statistically significant and clinically important.

Cisatracurium had the lowest rate of cross-reactivity in patients who had previously suffered anaphylaxis to rocuronium or vecuronium.

Anaphylaxis rates (highest to lowest)
Primary anaphylaxis: rocuronium > atracurium > vecuronium > pancuronium = cisatracurium
Cross-reactivity: suxamethonium > rocuronium > vecuronium > pancuronium > atracurium > cisatracurium

196
Q

22.2 A 25-year-old male has continued postoperative bleeding after an extraction of an impacted third molar tooth under a general anaesthetic. The patient mentions that his father bruises quite easily. His coagulation screen reveals: (provided). The most likely diagnosis is
(APTT raised, PT normal?)

a. Factor V leiden
b. haemophilia A
C. Von willebrand’s disease
D. Haemophilia B

A

b. von willebrand’s disease

  • autosomal dominant inheritance
  • may have normal or prolonged APTT, PT is normal

*Haem A: X-linked recessive disorder; would expect prolonged aPTT, and normal PT
*Haem B: X-linked recessive disorder; would expect normal aPTT and normal PT

REPEAT

vWD can have prolonged APTT or normal APTT. Haemophilias are X-linked

197
Q

22.1A 63-year-old woman is to undergo an elective total hip replacement. Her past medical history includes hypertension, stroke, type 2 diabetes mellitus, chronic atrial fibrillation and chronic renal impairment with an estimated creatinine clearance of 46 mL/min. Her medications include dabigatran 150 mg bd for stroke prevention. Perioperatively, her dabigatran therapy should

a. Be withheld 2 days
b. Withhold 3 days
c. Withhold 5 days
d. Withhold 6 days
e. Continue

A

5d

ANZCA - CrCl >80 (3D) 80-50 (4D) <50 (5D)

198
Q

22.2 A 6-year-old patient (140 cm, 24 kg, BSA 0.97m2) is on hydrocortisone 15 mg/day. Perioperative glucocorticoid supplementation is (considered if)

a.
b. Taking >1week
c. Taking >1 month
d. Taking >2 months
e. Taking >4 months

A

Taking > 1 month

https://associationofanaesthetists-publications.onlinelibrary.wiley.com/doi/full/10.1111/anae.14963

Daily doses of prednisolone of 5 mg or greater in adults and 10–15 mg.m−2 hydrocortisone equivalent or greater in children may result in hypothalamo–pituitary–adrenal axis suppression if administered for 1 month or more by oral, inhaled, intranasal, intra-articular or topical routes; this chronic administration of glucocorticoids is the most common cause of secondary adrenal suppression, sometimes referred to as tertiary adrenal insufficiency

All children who have known glucocorticoid deficiency (primary or secondary), or who are at risk of glucocorticoid deficiency (on significant exogenous dose of glucocorticoid >10–15 mg.m-2 per day) 38, should receive an i.v. dose of hydrocortisone at induction (2 mg.kg−1 for minor or major surgery under general anaesthesia).

199
Q

20.1 In a patient with known COPD, which of the following post bronchodilator spirometry results is consistent with a GOLD 3 classification? (Global initiative for chronic Obstructive Lung Disease)

a) FEV1 83%
b) FEV1 57%
c) FEV1 43%
d) FEV1 27%
e) FEV1 19%

A

c) FEV1 43%

In pulmonary function testing, a post-bronchodilator FEV1/FVC ratio of <0.70 is commonly considered diagnostic for COPD. The Global Initiative for Chronic Obstructive Lung Disease (GOLD) system categorises airflow limitation into stages. In patients with FEV1/FVC <0.70:

GOLD 1 - mild: FEV1 ≥80% predicted

GOLD 2 - moderate: 50% ≤ FEV1 <80% predicted

GOLD 3 - severe: 30% ≤ FEV1 <50% predicted

GOLD 4 - very severe: FEV1 <30% predicted.
200
Q

21.2 A 45-year-old man has the following results on his blood biochemistry testing: The most likely diagnosis is

  • Bili 30*
  • AST 1000*
  • ALT 500*
  • Albumin 30*
    *These blood results are not the original stem.

The most likely diagnosis is:

a) Hepatitis
b) Alcoholic liver disease
c) Paracetamol toxicity
d) Cholecystitis

A

b) Alcoholic liver disease
- AST>ALT

In hepatitis and paracetamol toxicity would expect ALT>AST.

In cholecystitis, would expect a cholestatic picture with raised conjugated bilirubin and raised GGT/ALP.

LITFL: Overall analysis of Liver Function Tests (LFT)

Transaminitis: Aminotransferases (AST, ALT)
- Generally associated with hepatocellular damage
- Generally not associated with cholestasis

Ratio of AST and ALT can be useful in differential
ALT is more specific for liver damage than AST

AST: ALT =1
-> Associated with ischaemia (CCF and ischaemic necrosis and hepatitis)

AST: ALT >2.5
-> Associated with Alcoholic hepatitis
-> Alcohol induced deficiency of pyridoxal phosphate
AST: ALT <1
-> High rise in ALT specific for Hepatocellular damage
-> Paracetamol OD with hepatocellular necrosis
-> Viral hepatitis, ischaemic necrosis, toxic hepatitis
-> Elevation with cholestasis (ALP, GGT)

ALP – primarily associated with cholestasis and malignant hepatic infiltration
Marker of rapid bone turnover and extensive bony metastasis

GGT – sensitive to alcohol ingestion
Marker of hepatocellular damage but non-specific
Sharpest rise associated with biliary and hepatic obstruction

201
Q

20.2 Severe obstructive sleep apnoea in adults is confirmed if during polysomnography if the apnoea/hypopnea index (AHI) is greater than or equal to

A) 10
B) 20
C) 30
D) 40
E) 50

A

C) 30

202
Q

22.2 A 72-year-old man with peripheral vascular disease presents for a femoral angioplasty and is currently taking aspirin. Regarding the perioperative management of his aspirin,

a) Cessation leads to increased risk of stroke
b) Cessation leads to increased risk of MI
c) Continuation leads to increased risk of major bleeding
d) Continuation leads to reduced rate of MI
e) Continuation leads to reduced rate of perioperative mortality

A

c) Continuation leads to increased risk of major bleeding

Aspirin in patients undergoing non cardiac surgery
https://www.nejm.org/doi/full/10.1056/nejmoa1401105

Conclusions

Administration of aspirin before surgery and throughout the early postsurgical period had no significant effect on the rate of a composite of death or nonfatal myocardial infarction but increased the risk of major bleeding. (Funded by the Canadian Institutes of Health Research and others; POISE-2 ClinicalTrials.gov number

203
Q

22.1 A 45-year-old woman is being assessed for liver transplantation. In order to determine the severity of her liver disease the Model for End-stage Liver Disease score is derived using the international normalised ratio, serum bilirubin and

a. GGT
b. Albumin
c. Sodium
d. ALT
e. Creatinine

A

Creatinine

Model for End-stage Liver Disease
- Estimates disease severity and survival in patients with Liver Disease
- Objective assessment
- Score from 6-40
- Validated across a number of liver diseases and surgeries
- MELD score is used as a method of allocation of organs to estimate survival

MELD uses the following parameters:
- Bilirubin
- INR
- Creatinine
- [Hyponatraemia]
○ Part of the MELD-Na score update in 2016
○ Sodium (Na) Values < 125 are set to 125 and values >137 are set to 137
4 MELD levels are:
- >/=25 (gravely ill)
- 24-19
- 18-11
- </=10

In patients who have undergone abdominal surgery an elevated MELD score was a better predictor of poor perioperative outcome than Child-Pugh Classification
- MELD score >15 should avoid elective surgery

Calculation:
MELD =
3.8loge(serum bilirubin [mg/dL]) + 11.2loge(INR) + 9.6*loge(serum creatinine [mg/dL]) + 6.4

MELD-Na =
MELD + 1.32 * (137-Na) - [0.033*MELD * (137-Na)]

204
Q

21.2 An awake patient in the post-anaesthesia care unit complains of breathlessness. The FiO2 is 0.4 via a facemask. An arterial blood gas taken at the time shows PaO2 135 mmHg, PaCO2 48 mmHg, and SpO2 100% The alveolar-arterial gradient (in mmHg) is
approximately

a) 60
b) 90
c) 120
d) 150

A

b) 90

PAO2: 0.4 (760 - 47) - 48/0.8 = 285 - 60 = 225mmHg
225 - 135 = 90mmHg.

205
Q

22.1 An adult male patient has a haemoglobin level of 80 g/L and his blood film shows a reticulocyte count of 10%. These findings are compatible with

a. ALL
b. Spherocytosis
c. Aplastic anaemia
d. Pernicious anaemia
e. Anaemia of chronic disease

A

Hereditary spherocytosis.

Auto-haemolytic, intraplenic haemolysis. High reticulocyte count (6-20%) (normal range 0.5-2%)

206
Q

21.1 Of the following, the LEAST likely cause of high anion gap metabolic acidosis is

a) lactic acidosis
b) renal failure
c) tuberculosis on isoniazid
d) renal tubular acidosis
e) salicylate overdose

A

renal tubular acidosis-> NAGMA

HAGMA:
Causes CATMUDPILES

- CO/CN
- Alcoholic ketoacidosis/ starvation ketoacidosis
- Toluene (paint thinners)
- Metformin/Methanol
- Uraemia
- DKA
- Pyroglutamic Acidosis/paracetamol/ phenformin/propylene glycol/paraldehyde
- Iron/ Isoniazid
- Ethylene glycol (anti-freeze)
- Salicylic Acid

NAGMA:
Causes CAGE:
- Chloride
- Acetazolamide and Addison’s
- GI causes (vomiting, diarrhoea, fistula)
Extras: RTA

207
Q

22.1 A 45-year-old woman is reviewed in the preadmission clinic. She is scheduled to undergo a microwave endometrial ablation for menorrhagia in one week’s time. Her preoperative laboratory investigations include the following blood results (full blood examination and iron studies shown).
The most appropriate course of action would be to

a. Proceed
b. Iron IV then proceed
c. Transfuse 2 RBC intraop
d. Use cell saver intraop
e. Defer and refer to haematology for further Ix

A
208
Q

22.2 AFE incidence highest in
a) HELP Syndrome
b) LSCS
c) Instrumental delivery
d) Preeclampsia
e) Spontaneously

A

LUCS

209
Q

22.1 A 72-year-old female smoker with hypertension presents to the emergency department with a wrist fracture after a fall. She has been increasingly tired and confused over the previous week. Her serum and urine electrolytes are (supplied). The most likely diagnosis is

(Low K, low Na, Normal Ur and Cr, Ur sodium <10mmol/L)

a. SIADH
b. Addison’s
c. Diuretic

A

Uncertain

SIADH:
1. hypotonic hyponatraemia
2. urine osmolality > plasma osmolality (<275mOsm/kg) (i.e. concentrated urine despite hypotonic blood)
3. urinary Na+ > 20mmol/L
4. normal renal, hepatic, cardiac, pituitary, adrenal and thyroid function
5. euvolaemia (absence of hypotension, hypovolaemia, and oedema)
6. correction by water restriction

Addison’s
Hypo natraemia
HYPER kalaemia
Hypo glycaemia
Acidosis

Diuretics
Hypo natraemia
Hypo kalaemia
High urinary Na and K

https://www.derangedphysiology.com/files/Electrolyte%20Disturbance.pdf

JAMA article on hyponatraemia
https://emergencymed.org.il/wp-content/uploads/2022/08/jama_adrogu_2022_rv_220011_1657919726.49616.pdf

210
Q

21.1 Severe obstructive sleep apnoea in a 6-year-old child is confirmed if during polysomnography the
apnoea/hypopnea index (AHI) is greater than or equal to

A >5
B >10
C >15
D >20
E >30

A

> 10

211
Q

21.2 The diffusing capacity of the lungs for carbon monoxide (DLCO) is likely to be decreased with

a) Sarcoidosis
b) Asthma
c) Obesity
d) Pulmonary haemorrhage

A

sarcoid

212
Q

22.2 A 55-year-old man with no past history of ischaemic heart disease is three days post-total hip replacement surgery. He has an episode of chest pain that sounds ischaemic, began at rest and lasts 30 minutes before resolving fully. There are no ECG changes nor troponin rise. The diagnosis is

a. No diagnosis made
b. Unstable angina
c. STEMI
d. NSTEMI
e. MINS

A

b. Unstable angina

UTD:

Unstable angina (UA) and acute non-ST elevation myocardial infarction (NSTEMI) differ primarily in whether the ischemia is severe enough to cause sufficient myocardial damage to release detectable quantities of a marker of myocardial injury (troponins):

●UA is considered to be present in patients with ischemic symptoms suggestive of an ACS and no elevation in troponins, with or without electrocardiogram changes indicative of ischemia (eg, ST segment depression or transient elevation or new T wave inversion).

●NSTEMI is considered to be present in patients having the same manifestations as those in UA, but in whom an elevation in troponins is present.

MINS: Myocardial injury after non-cardiac surgery (up to 30 days post-op):
1. Elevated postop troponin
2. Resulting from myocardial ischaemia (i.e. no evidence of a non-ischaemic aetiology), not requiring an ischaemic feature (i.e. no chest pain, no ECG change)

VISION studies (Vascular Events in Noncardiac Surgery Patients Cohort Evaluation) demonstrated that severity of MINS strongly associated with 30-day mortality after NCS.

hs-cTnT
<20ng/L ~ 0.5% 30 day mortality
20-64ng/L ~3% 30 day mortality
65-999 ng/L ~9% 30 day mortality
>1000ng/L ~30% 30 day mortality

Whilst VISION trial identified MINS in at risk patients, the question now becomes what interventions are available to prevent this complication?

213
Q

20.2 The flow volume loop is most consistent with (Flow-volume loop shown)

a) Variable intra-thoracic obstruction
b) Variable extra-thoracic obstruction
c) Lower airway obstruction
d) Restrictive lung pattern
e) Mixed pattern

A

c) Lower airway obstruction

Peripheral or lower airways obstruction: expiratory limb demonstrates concave upward, also called “scooped-out” or “coved” pattern.

214
Q

20.2 International guidelines state that patients presenting for major surgery have inadequate or low iron stores if their serum ferritin level is less than

a. 20 mcg/L
b. 30 mcg/L
c. 40 mcg/L
d. 50 mcg/L
e. 100 mcg/L

A

E. 100 mcg/L

?? < 30mcg/L

< 100mcg/L IF CRP > 5 and/or Transferrin saturation < 20

https://associationofanaesthetists-publications.onlinelibrary.wiley.com/doi/10.1111/anae.13773#:~:text=Recommendations%20for%20best%20clinical%20practice,-Physicians%20should%20consider&text=The%20presence%20of%20anaemia%20should,identification%20of%20absolute%20iron%20deficiency.

215
Q

22.2 The curve labelled ‘b’ is most likely to represent the flow–volume loop of a patient with

a) Asthma
b) Post lung transplant
c) Pulmonary fibrosis
d) Tracheal stenosis
e) VC palsy

A

Tracheal stenosis

216
Q

20.1 In patients with sepsis and acute kidney injury, early renal replacement therapy (<12 hours) compared to a delayed strategy (48 hours) results in

a) Decreased 90 day mortality
b) No difference
c) Decreased icu time
d) Decreased length of admission
e) Increased 90 day mortality

A

b) No difference

NEJM: Timing of Renal-Replacement Therapy in Patients with Acute Kidney Injury and Sepsis

Among patients with septic shock who had severe acute kidney injury, there was no significant difference in overall mortality at 90 days between patients who were assigned to an early strategy for the initiation of renal-replacement therapy and those who were assigned to a delayed strategy.

217
Q

22.1 According to the ‘Fourth Consensus Guidelines for the Management of Postoperative Nausea and Vomiting (PONV)’ published in 2020, multimodal PONV prophylaxis should be implemented in adult patients

a. For everyone
b. 1 or more RF
c. 2 or more RF
d. 3 or more RF
e. 4 or more RF

A

b. 1 or more RF

In this iteration of the PONV guideline, one of the major changes is that we now recommend the use of multimodal prophylaxis in patients with one or more risk factors. This decision was made due to the concern over inadequate prophylaxis as well as the availability of antiemetic safety data.

218
Q

21.1 The substance that should be avoided in a patient with history of anaphylaxis to MMR vaccine is

a. Protamine
b. Gelofusine
c. Sulphonamides
d. Penicilins

A

b. Gelofusin
Gelatin

219
Q

23.1 The use of erythropoietin before major surgery results in

a) Less transfusion, same thrombosis
b) Less transfusion, more thrombosis
c) No change in transfusion or thrombosis
d) No change in transfusion, more thrombosis

A

repeat

a) Less transfusion, same thrombosis

●A 2019 meta-analysis of randomized trials comparing preoperative administration of EPO versus placebo (32 trials; 4750 patients, mostly orthopedic and cardiac surgery) found reduced blood transfusions in the EPO groups. Decreased blood transfusions were seen in the entire population (RR 0.59, 95% CI 0.47-0.73; 28 trials), as well as the subgroups undergoing cardiac surgery (RR 0.55, 95% CI 0.47-0.73; nine trials) and major orthopedic surgery (RR 0.36, 95% CI 0.28-0.46; five trials). In addition, the EPO group had increased hemoglobin levels. There was no increase in the incidence of thromboembolic events with EPO.

220
Q

23.1 The following pressure-volume loop is displayed on your ventilator screen. The
shape of this loop indicates

a. Over-expansion
b. Under-expansion
c. Normal ventilation
d. PEEP too high
e. PEEP too low

A

a) over-expansion

The first graphic (loop a) shows the pattern of a typical pressure-volume loop, which rises in a counterclockwise direction until forming a complete loop. It also displays inflection points, which display rapid changes to the slope of the limb.

The lower inflection point (LIP) occurs due to the opening of collapsed alveoli, resulting in a sharp increase in volume. The upper inflection point (UIP) occurs near the end of inspiration when more pressure leads to only a minimal increase in volume.

The second graphic (loop b) displays how overdistension and hysteresis appear on a pressure-volume loop. Overdistention occurs when the lungs receive too much volume or pressure and can result in injury. Hysteresis refers to lung tissue that behaves differently on inspiration and expiration.

In other words, it takes more energy for the lungs to inflate than it does to deflate. Therefore, hysteresis on a pressure-volume loop refers to the space between the inspiratory and expiratory limbs. When the patient’s lung compliance or airway resistance changes, so will the hysteresis and, thus, the appearance of the loop.

Note: A pressure-volume loop under normal conditions should resemble the shape of a football (American). A curve with a flat appearance indicates decreased lung compliance. A steep curve, on the other hand, indicates increased lung compliance. A wide curve indicates increased airway resistance, whereas the opposite is true if the loop appears more narrow.

https://www.respiratorytherapyzone.com/ventilator-waveforms/#:~:text=Note%3A%20A%20pressure%2Dvolume%20loop,hand%2C%20indicates%20increased%20lung%20compliance.

221
Q

23.1 One metabolic equivalent (1 MET) is defined as the

a. O2 consumption walking 4km/h
b. O2 consumption when sitting
c. Energy expenditure walking 4km/h
d. Energy expenditure when sitting.

A

b) O2 consumption when sitting

One metabolic equivalent (MET) is defined as the amount of oxygen consumed while sitting at rest and is equal to 3.5 ml O2 per kg body weight x min.

222
Q

22.1 According to the Third International Consensus Definitions for Sepsis and Septic Shock (Sepsis-3), sepsis is defined as

a. SIRS criteria
b. Life threaning organ dysfunction with vasopressor requirement to maintain MAP >65 and lactate >2
c. Life threatening organ dysfunction caused by a dysregulated host response to infection
d. sBP <100, RR>22, altered mentation

A

Life threatening organ dysfunction caused by a dysregulated host response to infection

223
Q

23.1 A patient’s glomerular filtration rate is estimated at 35 mL/min/1.73m2. The patient’s chronic kidney disease can be classified as Stage

a. 5
b. 4
c. 3a
d. 3b
e. 2

A

Category GFR
ml/min/1.73 m2 Terms
G1 ≥90 Normal or high
G2 60-89 Mildly decreased*
G3a 45-59 Mildly to moderately decreased
G3b 30-44 Moderately to severely decreased
G4 15-29 Severely decreased
G5 <15 Kidney failure

Assign Albuminuria category as follows:
Albuminuria categories in CKD
Category ACR (mg/g) Terms
A1 <30 Normal to mildly increased
A2 30-300 Moderately increased*
A3 >300 Severely increased**
Abbreviations: ACR, albumin-to-creatinine ratio; CKD, chronic kidney disease.
*Relative to young adult level.
**Including nephrotic syndrome (albumin excretion ACR >2220 mg/g)

**Collectively referred to as “CGA Staging”

REPEAT

224
Q

21.2 International guidelines state that patients presenting for major surgery have inadequate or low iron stores if their serum ferritin level is less than

a) 20
b) 30
c) 50
d) 100

A

ANSWER: d. Ferritin <100mcg/L

Serum ferritin level < 30 μg.l−1 is the most sensitive and specific test used for the identification of absolute iron deficiency. However, in the presence of inflammation (C-reactive protein > 5 mg.l−1) and/or transferrin saturation < 20%, a serum ferritin level < 100 μg.l−1 is indicative of iron deficiency.

International consensus statement on peri-operative management of anaemia and iron deficiency

https://associationofanaesthetists-publications.onlinelibrary.wiley.com/doi/10.1111/anae.13773#:~:text=Recommendations%20for%20best%20clinical%20practice,-Physicians%20should%20consider&text=Serum%20ferritin%20level%20%3C%2030%20%CE%BCg,serum%20ferritin%20level%20%3C%20100%20%CE%BCg.

225
Q

20.1 70 year old patient for revision THR, in clinic 10 days prior

Hb 110
Ferritin 51
CRP 10
What should you do?

a Transfuse 2u pRBC
b Give oral iron therapy and continue with surgery
c Give oral iron therapy and defer surgery for 6 weeks
d Give IV iron
e Do nothing

A

c Oral iron and defer

or

d give IV iron

  • most assume its IV iron and proceed but
  • Assuming IDA and raised CRP then iron therapy but
    ‘deferable’ surgery? then oral and come again in 6 weeks
    if not deferrable then IV iron - surely a revision THR is deferable??
  • If give IV iron and defer was an option I would choose that one, it would allow assessment of inflammatory process and to confirm Hb and ferritin are at an acceptable level

International consensus statement on the peri-operative management of anaemia and iron deficiency

https://associationofanaesthetists-publications.onlinelibrary.wiley.com/doi/10.1111/anae.13773

However, many patients will not respond to oral iron, especially those with functional iron deficiency and chronic illness or infection and those with ongoing blood loss 15, 25. Others will not tolerate oral iron due to gastro-intestinal side-effects. Once oral iron has been commenced, the Hb should be measured again, at least 4 weeks before surgery. In the absence of an increased Hb or if the patient is intolerant, i.v. iron is the preferred replacement route. If surgery is planned in less than 6 weeks time, i.v. iron may also be the most effective option.

226
Q

22.2 A raised (> 140% predicted) single-breath diffusing capacity of the lung for carbon monoxide (DLCO) can be caused by

a. Emphysema
b. COPD
c. interstitial lung disease
d. Asthma
e. Sarcoidosis

A

d. Asthma

What are the causes of an elevated DL CO ?

The causes of an elevated DLCO are numerous, but is most commonly caused by asthma and obesity (increased pulmonary blood flow). Pulmonary hemorrhage is an additional important cause.

https://www.atsjournals.org/doi/pdf/10.1513/AnnalsATS.201605-355CC

227
Q

21.2 A 50 year old man has the following pulmonary function test result:

FEV1 98% predicted
FVC 95% predicted
DLCO 43% predicted

The diagnosis is most consistent with:

a) Pulmonary fibrosis
b) Pulmonary hypertension
c) COPD
d) Obesity

A

b) Pulmonary hypertension

Up to date: Overview of pulmonary function testing in adults

Diffusing capacity — Measurement of the single-breath diffusing capacity for carbon monoxide (DLCO; also known as transfer factor or TLCO) is quick, safe, and useful in the evaluation of restrictive and obstructive lung disease, as well as pulmonary vascular disease. The technique and interpretation are discussed separately.

In the setting of restrictive disease, the diffusing capacity helps distinguish between intrinsic lung disease, in which DLCO is usually reduced, and other causes of restriction, in which DLCO is usually normal.

In the setting of obstructive disease, the DLCO helps distinguish between emphysema, in which it is usually reduced, and other causes of chronic airway obstruction, like asthma or chronic bronchitis, where it is usually normal.

The DLCO is also used in the assessment of pulmonary vascular disease (eg, thromboembolic disease, pulmonary hypertension), which typically causes a reduction in DLCO in the absence of significant restriction or obstruction

228
Q

22.1 Anaesthesia-induced rhabdomyolysis differs from malignant hyperthermia in that it is NOT

a. Reduced myoglobinuria
b. Less increase in ETCO2
c. Less muscle rigidity

A

a. Reduced myoglobinuria

weird double negative question, answers don’t quite fit grammatically, however
- There is NOT reduced myoglobinuria with AIR compared to MH (both have myoglobinuria)
- There IS less increase in ETCO2
- There IS less muscle rigidity

Anesthesia-induced rhabdomyolysis or malignant hyperthermia: is defining the crisis important?
College library drop “is defining the crisis important?” from search
https://onlinelibrary-wiley-com.ezproxy.anzca.edu.au/doi/full/10.1111/pan.13130?sid=worldcat.org

229
Q

22.2 After ceasing smoking, a patient’s immune function has effectively recovered to normal after

a) 1 day
b) 3 weeks
c) 6-8 weeks
d) 6 months
e) 6 years

A

d) 6 months

ANZCA PS 12 perioperative smoking
https://www.anzca.edu.au/getattachment/5deb6800-e8f9-453f-b9a6-a151a9323249/PG12(POM)-Guideline-on-smoking-as-related-to-the-perioperative-period-(PS12)

Effects of quitting
1 day
- Reduced HbCO3-> increased O2 content
- Reduced nicotine/ SNS stimulation

3 weeks
- Increased wound healing

6-8 weeks
- Reduced sputum volume
- Increased lung function

6 months
- Increased immune function

230
Q

20.1 Patients with obstructive sleep apnoea undergoing surgery, have been shown to have an increased incidence of

A) AF
B) Acute renal failure
C) AMI
D) Perioperative mortality
E) Unplanned admission after ambulatory surgery

A

C) AMI

The association between OSA and perioperative mortality is unclear [19], with various studies showing increased [12,23] or comparable [14] mortality. While decreased mortality [7,18,43] has been reported in retrospective studies, this may be due to unrecognized or undiagnosed OSA in the control or non-OSA groups, and/or the possibility that patients with diagnosed OSA received monitoring and treatment of their OSA.

SOBA 2015:
Severe OSA occurs in 10–20% of patients with BMI > 35 kg.m2 and is often undiagnosed. Overall, a diagnosis of OSA is associated with a greater than doubling of the incidence of postoperative desaturation, respiratory failure, postoperative cardiac events and ICU admission [16]. The presence of multiple and prolonged oxygen desaturations increases the sensitivity to opioid-induced respiratory depression [17]. However, if identified pre-operatively and treated appropriately with continuous positive airway pressure (CPAP), the risk of complications is much reduced [18].

Obesity has no influence on the rate of unanticipated admission, postoperative complications, readmission or other unplanned contact with health professionals after home discharge.

231
Q

21.2 A derived value from an arterial blood gas sample is

a) PaO2
b) PaCO2
c) pH
d) BE

A

HCO3- is derived from pCO2 and pH
Base excess is derived from pH
SaO2 is derived from oxyHb and Hb

Source LITFL

232
Q

20.2 An ASA 1 28 year old male attends for inguinal hernia repair under general anaesthesia. He is administered propofol 180mg morphine 8mg rocuronium 50mg cephazolin 2g Post induction he develops an erythematous rash on his chest and arms, swelling of his lips and face, and severe hypotension. Preliminary blood results show …

Elevated tryptases (100 -> 40)
normal Ig E level
elevated morphine RAST.

The most likely diagnosis is

a) Ig E mediated morphine allergy
b) IgE mediated rocuronium allergy
c) Morphine induced histamine release
d) IgE mediated cephazolin allergy
e) Mastocytosis

A

b) IgE mediated (i.e. anaphylaxis) rocuronium allergy

Morphine RAST is most sensitive (88%) and specific (100%) test for NMBD as cause of anaphylaxis (quaternary ammonium epitope)

233
Q

The Sequential Organ Failure Assessment (SOFA) score is used in intensive care for the
assesment of sepsis. This score does NOT include the:

a) MAP
b) FiO2/PaO2
c) INR
d) GCS
e) Plts

A

Previous Q (23.1) with different options.
ANSWER C (INR)

234
Q

The modified Aldrete scoring system uses all of the following EXCEPT the

a) BP
b) Pain score
c) Resp rate
d) sedation level

A

b) Pain score

Aldrete score, which includes five elements (activity, respiration, circulation, consciousness, oxygen saturation) [16].

The original scoring system was developed before the invention of pulse oximetry and used the patient’s colouration as a surrogate marker of their oxygenation status. A modified Aldrete scoring system was described in 1995 which replaces the assessment of skin colouration with the use of pulse oximetry to measure SpO2.

The Modified Aldrete system includes five additional elements that are particularly useful during the Phase II recovery period prior to discharge to home (dressing, pain, ambulation, feeding, urine output)

235
Q

A patient’s glomerular filtration rate is estimated at 35 mL/min/1.73m2. The patient’s chronic kidney disease can be classified as Stage
a) 2
b) 3a
c) 3b
d) 4
e) 5

A

c) 3b

236
Q

Diffusing capacity of the lungs for carbon monoxide (DLCO) is decreased in all of the following EXCEPT

made up potential answers:

a) Pulmonary Fibrosis
b) Interstitial Lung disease
c) Obesity
d) Pulmonary haemorrhage

A

d) Pulmonary haemorrhage

Rewording of 21.2 Question

Won’t increase in Myasthenia Gravis

Causes of HIGH value include:
Asthma
Left-right intracardiac shunt
polycythaemia
Pulmonary haemorrhage
Obesity - Dlco will increase but kco will not

237
Q

With regard to Donation after Circulatory Determination of Death (DCDD), the maximum
acceptable time from withdrawal of cardio-respiratory support to cold perfusion for liver
donation is

a) 30mins
b) 45 mins
c) 60 mins
d) 90 mins

A

Warm ischaemia time:
- Time from treatment withdrawal to the start of cold perfusion of the donated organs
- Significance is the impact on graft function
- Most important phase of WIT begins when the systolic BP is < 60mmHg
- This includes the waiting period from the absence of circulation to the declaration of death and the time before initiating the flow of cold perfusate through the cannula

Maximum WARM Ischaemia time
- Heart 30 mins
- Liver 30 mins
- Pancreas 30 mins
- Kidney 60 mins
- Lungs 90 mins

Maximum COLD Ischaemia time:
- Heart = 4 hrs
- Lungs = 6-8hrs
- Liver/Pancreas = 12hrs (DBD)/6 hrs (DCD)
- Kidneys = 18hrs (DBD)/ 12 hrs (DCD)

238
Q

During a thyroidectomy, the surgeon is concerned the parathyroid glands have been
devascularised. From the time of potential damage, a serum calcium level should be checked in

a) 6hrs
b) 12hrs
c) 24 hrs
d) 36hrs

A

24hrs

Oxford handbook

239
Q

One metabolic equivalent (MET) is equal to

a. O2 consumption walking 4km/h
b. O2 consumption when sitting
c. Energy expenditure walking 4km/h
d. Energy expenditure when sitting.

A

b) O2 consumption when sitting

One metabolic equivalent (MET) is defined as the amount of oxygen consumed while sitting at rest and is equal to 3.5 ml O2 per kg body weight x min.

240
Q

Refeeding syndrome following the commencement of total parenteral nutrition is associated with the development of

A

Most likely answer will be related to hypophosphataemia

Refeeding syndrome is a constellation of biochemical abnormalities which occurs when normal intake is resumed after a period of starvation. Its characteristic features are low levels of phosphate, potassium, magnesium and sodium. Its major complications include cardiac arrhythmias, heart failure (due to hypophosphataemia), muscle weakness, rhabdomyolysis, seizures and an altered sensorium.

The major risk factors are calorie malnutrition of any cause, alcohol or drug use, low BMI (18-16) and starvation for 5-10 days.

Pathophysiology
With the restoration of glucose as a substrate, insulin levels rise and cause cellular uptake of these ions. Depletion of adenosine triphosphate (ATP) and 2,3-diphosphoglyceric acid (2,3-DPG) results in tissue hypoxia and failure of cellular energy metabolism. This may manifest as cardiac and respiratory failure, with paraesthesiae and seizures also reported. Thiamine deficiency may also play a part.

  • Exogenous sources of phosphate are inadequate to supplement the daily phosphate requirements
  • Intracellular phosphate stores are used to synthesise ATP (using protein and fat as fuel)
  • Homeostatic mechanisms maintain serum concentrations of these ions at the expense of intracellular stores

Reference: https://derangedphysiology.com/main/required-reading/endocrinology-metabolism-and-nutrition/Chapter%20315/refeeding-syndrome “

241
Q

A patient taking tranylcypromine, a monoamine oxidase inhibitor, requires elective surgery.
The best management is to

(made up answers)

a) Cease 1 month before surgery
b) Do not Cease
c) Cease day of surgery
d) Cease 2 weeks before surgery
e) stop 2 weeks before, start moclobemide and omit Moclobemide day of surgery

A

e) stop 2 weeks before, start moclobemide and omit Moclobemide day of surgery
-> probably in discussion with the patients psychiatrist

Tranylcypromine, sold under the brand name Parnate among others, is a monoamine oxidase inhibitor. More specifically, tranylcypromine acts as nonselective and irreversible inhibitor of the enzyme monoamine oxidase.

In the elective setting, there is some debate regarding the management of patients on MAOI. Although the risks associated with anaesthesia in those taking this group of drugs are significant, abrupt withdrawal may precipitate serious psychiatric relapse. Traditionally, irreversible MAOIs have been stopped 2 weeks before operation; however, omitting the dose of moclobemide on the day of surgery is acceptable. It has been suggested that in the elective situation, patients could be switched from an irreversible MAOI to moclobemide to avoid a prolonged period of discontinuation.

242
Q

A 46-year-old woman with menorrhagia is booked for abdominal hysterectomy. Her
preoperative bloods show

creatinine 55
Ca2+ 2.2
PO43- 0.34.

The most likely reason for these findings is

a) Diuretic use
b) Fanconi syndrome
c) Hyperparathyrodisim
d) Vit D deficiency
a) Iron transfusion

A

a) Iron transfusion

Iron infusion (ferric carboxymaltose) – can cause renal wasting of phosphate resulting in severe hypophosphataemia

Vitamin D deficiency and hyperparathyroidism can also cause hypophosphataemia. Vitamin D deficiency would result in low calcium whereas hyperparathyroidism would result in hypercalcaemia.
Fanconi syndrome: rare defect of proximal tubule leading to decreased reabsorption -> results in hypokalaemia, hypophosphataemia, hyperchloraemic metabolic acidosis. https://www.ncbi.nlm.nih.gov/pmc/articles/PMC6689119/